Derivada de una funciĂ³n en un punto del dominio de la misma

IntroducciĂ³n

La derivada de una funciĂ³n sirve para formalizar la nociĂ³n de recta tangente en un punto \(x_0\) para una funciĂ³n \(f(x)\).

En el grĂ¡fico siguiente, podemos observar un ejemplo del dibujo de una funciĂ³n junto con la recta tangente en un punto \(x_0\) de su dominio.

La pendiente de la recta tangente (en rojo) vendrĂ­a a representar lo que denominaremos la derivada de la funciĂ³n \(f(x)\) en \(x=x_0\).

IntroducciĂ³n

IntroducciĂ³n

DefiniciĂ³n de derivada de una funciĂ³n en un punto

Consideremos la figura anterior.

La pendiente de una recta recordemos que se define como la tangente del Ă¡ngulo con el eje de las \(x\).

Hemos dibujado una funciĂ³n \(f(x)\) que pasa por el punto \((x_0,f(x_0))\) junto con una recta secante que pasa por el punto.

Otro punto de dicha recta dado un valor \(h\) serĂ­a \((x_0+h,f(x_0+h))\). Intuitivamente dicha recta secante “tiende” a la tangente cuando el valor \(h\) tiende a cero.

DefiniciĂ³n de derivada de una funciĂ³n en un punto

La pendiente de la recta secante como puede observarse es la tangente del Ă¡ngulo \(\alpha\) que vale \(\frac{f(x_0+h)-f(x_0)}{h}\). Entonces, la definiciĂ³n de derivada de \(f(x)\) en \(x=x_0\) es la siguiente:

DefiniciĂ³n de derivada de una funciĂ³n en un punto

DefiniciĂ³n de derivada en un punto.

Sea \(f:(a,b)\longrightarrow \mathbb{R}\) una funciĂ³n real de variable real. Sea \(x_0\in (a,b)\). Diremos que \(f\) es derivable en \(x_0\) o que existe la derivada de \(f\) en \(x_0\) cuando existe el lĂ­mite siguiente: \[ \lim_{h\to 0}\frac{f(x_0+h)-f(x_0)}{h}, \] y, en caso en que exista, llamaremos a dicho lĂ­mite derivada de la funciĂ³n \(f\) en \(x_0\) escrita matemĂ¡ticamente como \(f'(x_0)\).

DefiniciĂ³n de derivada de una funciĂ³n en un punto

ObservaciĂ³n.

La derivada de una funciĂ³n es un propiedad local, es decir, estĂ¡ definida en un punto \(x_0\) del dominio.

Cuando, haciendo un abuso del lenguaje, se dice que la funciĂ³n \(f\) es derivable, se quiere decir que dicha funciĂ³n es derivable en todos los puntos del dominio de la misma.

DefiniciĂ³n de derivada de una funciĂ³n en un punto

ObservaciĂ³n.

La derivada de una funciĂ³n \(f\) en un punto de su dominio \(x_0\) se puede expresar tambiĂ©n de la forma siguiente: \[ \lim_{x\to x_0}\frac{f(x)-f(x_0)}{x-x_0}. \] Para comprobar la afirmaciĂ³n anterior, basta considerar el cambio de variable siguiente \(x=x_0+h\), de donde \(h=x-x_0\) y aplicar la definiciĂ³n de derivada.

Fijaos que decir que \(h\) tiende a cero es equivalente a decir que \(x\) tiende a \(x_0\).

Ejemplos

Derivada de la funciĂ³n constante

Veamos que si la funciĂ³n \(f\) es constante en todo su dominio, \(f(x)=k\), para todo \(x\) del dominio, la derivada de \(f\) en cualquier punto del mismo es nula.

Sea \(x_0\) un punto del dominio de \(f\). La derivada de \(f\) en \(x_0\) serĂ¡: \[ f'(x_0)=\lim_{x\to x_0}\frac{f(x)-f(x_0)}{x-x_0}=\lim_{x\to x_0}\frac{k-k}{x-x_0}=\lim_{x\to x_0} 0 =0. \]

Ejemplos

Derivada del monomio de grado \(n\)

Calculemos la derivada de funciĂ³n \(f\) si Ă©sta vale \(f(x)=x^n\), donde \(n\) es un valor natural mayor que \(1\) (\(n=1,2,\ldots\)):

Sea \(x_0\) un punto del dominio de \(f\). La derivada de \(f\) en \(x_0\) serĂ¡: \[ \begin{array}{rl} f'(x_0) & =\displaystyle\lim_{x\to x_0}\frac{f(x)-f(x_0)}{x-x_0}=\lim_{x\to x_0}\frac{x^n-x_0^n}{x-x_0}=\lim_{x\to x_0} \frac{(x-x_0)(x^{n-1}+x^{n-2}x_0+x^{n-3}x_0^2+\cdots+ x_0^{n-1})}{x-x_0} \\ & \displaystyle =\lim_{x\to x_0} x^{n-1}+x^{n-2}x_0+x^{n-3}x_0^2+\cdots+ x_0^{n-1} =n\cdot x_0^{n-1}. \end{array} \]

Ejemplo

Derivada del monomio de grado \(n\)

La derivada de la funciĂ³n anterior en Wolfram Alpha se muestra en el enlace siguiente:

Ejemplos

Derivada del valor absoluto

Consideremos la funciĂ³n \(f(x)=|x|\), valor absoluto de \(x\), definida en todo \(\mathbb{R}\) como: \(f(x)=\begin{cases} x, & \mbox{si } x\geq 0, \\ -x & \mbox{si }x<0. \end{cases}\)

Estudiemos la derivabilidad de \(f\) en \(x_0=0\), es decir, veamos si el lĂ­mite siguiente existe: \(\displaystyle\lim_{x\to 0}\frac{f(x)-f(0)}{x-0}=\lim_{x\to 0}\frac{|x|}{x}\).

Si hacemos el lĂ­mite anterior por la derecha o para los valores \(x>0\), obtenemos: \[ \lim_{x\to 0^+}\frac{|x|}{x}=\lim_{x\to 0^+}\frac{x}{x}=\lim_{x\to 0^+}1 =1. \] En cambio, si lo hacemos por la izquierda o para los valores \(x<0\), obtenemos: \[ \lim_{x\to 0^-}\frac{|x|}{x}=\lim_{x\to 0^-}\frac{-x}{x}=\lim_{x\to 0^-} -1 =-1. \]

Ejemplos

Derivada del valor absoluto

Como los lĂ­mites anteriores no coinciden, concluimos que el lĂ­mite \(\displaystyle \lim_{x\to 0}\frac{|x|}{x}\) no existe y por tanto, \(f\) no es derivable en \(x=0\).

GrĂ¡ficamente se observa que la funciĂ³n anterior tiene una punta en \(x=0\) y, por tanto, \(f\) no puede ser derivable en dicho punto.

Este comportamiento es el usual cuando una funciĂ³n no es derivable en un punto \(x_0\), es decir, se observa grĂ¡ficamente que en dicho punto, la funciĂ³n no tiene un comportamiento suave.

Ejemplos

Derivada del valor absoluto

Ejemplo

Derivada del valor absoluto

La grĂ¡fica del valor absoluto en Wolfram Alpha se muestra en el enlace siguiente:

Propiedades de las funciones derivables

Derivabilidad implica continuidad

Teorema.

Sea \(f\) una funciĂ³n real de variable real y sea \(x_0\) un punto del dominio de \(f\). Si \(f\) es derivable en \(x_0\), entonces \(f\) es continua en \(x_0\).

Derivabilidad implica continuidad

DemostraciĂ³n.

Definimos la funciĂ³n siguiente en un entorno de punto \(x_0\): \[ g(x)=\begin{cases} \frac{f(x)-f(x_0)}{x-x_0}, & \mbox{si }x\neq x_0,\\ f'(x_0), & \mbox{si } x=x_0. \end{cases} \] Usando que \(f\) es derivable en \(x_0\), tenemos que la funciĂ³n \(g\) definida anteriormente serĂ¡ continua en \(x_0\) ya que: \[ \lim_{x\to x_0} g(x)=\lim_{x\to x_0} \frac{f(x)-f(x_0)}{x-x_0}=f'(x_0)=g(x_0). \] Si despejamos \(f(x)\) de la expresiĂ³n de \(g(x)\) obtenemos \(f(x)=f(x_0)+g(x)\cdot (x-x_0)\).

Veamos que \(f\) es continua en \(x_0\): \[ \lim_{x\to x_0}f(x)=\lim_{x\to x_0} f(x_0)+g(x)\cdot (x-x_0) = f(x_0)+f'(x_0)\cdot 0 = f(x_0), \] tal como querĂ­amos ver.

La derivada de la suma es suma de derivadas

ProposiciĂ³n.

Sean \(f\) y \(g\) dos funciones reales de variable real y sea \(x_0\) un valor del dominio de \(f\) y de \(g\). Si \(f\) y \(g\) son derivables en \(x_0\), tambiĂ©n lo es la funciĂ³n suma \(f+g\), y se verifica que: \((f+g)'(x_0)=f'(x_0)+g'(x_0)\).

DemostraciĂ³n: \[ \begin{array}{rl} (f+g)'(x_0) & = \displaystyle\lim_{x\to x_0}\frac{(f+g)(x)-(f+g)(x_0)}{x-x_0}=\lim_{x\to x_0}\frac{f(x)+g(x)-f(x_0)-g(x_0)}{x-x_0} \\ & = \displaystyle \lim_{x\to x_0}\frac{f(x)-f(x_0)}{x-x_0} + \lim_{x\to x_0}\frac{g(x)-g(x_0)}{x-x_0}=f'(x_0)+g'(x_0). \end{array} \]

La derivada de una funciĂ³n por una constante es la constante por la derivada de la funciĂ³n

ProposiciĂ³n.

Sea \(k\in\mathbb{R}\) un valor real, \(f\) una funciĂ³n real de variable real y \(x_0\) un punto del dominio de \(f\). Entonce si \(f\) es derivable en \(x_0\), tambiĂ©n lo es \(k\cdot f\) y se verifica que: \((k\cdot f)'(x_0)=k\cdot f'(x_0)\).

DemostraciĂ³n:

\[ \begin{array}{rl} (k\cdot f)'(x_0) & \displaystyle =\lim_{x\to x_0}\frac{(k\cdot f)(x)-(k\cdot f)(x_0)}{x-x_0}= \lim_{x\to x_0}\frac{k\cdot f(x)-k\cdot f(x_0)}{x-x_0}=\lim_{x\to x_0}\frac{k\cdot (f(x)-f(x_0))}{x-x_0} \\ &\displaystyle =k\lim_{x\to x_0}\frac{ f(x)-f(x_0)}{x-x_0}=k f'(x_0). \end{array} \]

La derivada del producto de funciones

ProposiciĂ³n.

Sean \(f\) y \(g\) dos funciones reales de variable real y sea \(x_0\) un punto del dominio de \(f\) y de \(g\). Si \(f\) y \(g\) son derivables en \(x_0\), tambiĂ©n lo es la funciĂ³n producto \(f\cdot g\), y se verifica que: \((f\cdot g)'(x_0)=f'(x_0)\cdot g(x_0)+f(x_0)\cdot g'(x_0)\).

DemostraciĂ³n: \[ \begin{array}{rl} (f\cdot g)'(x_0) & = \displaystyle\lim_{x\to x_0}\frac{(f\cdot g)(x)-(f\cdot g)(x_0)}{x-x_0}=\lim_{x\to x_0}\frac{f(x)\cdot g(x)-f(x_0)\cdot g(x_0)}{x-x_0} \\ & = \displaystyle \lim_{x\to x_0}\frac{f(x)\cdot g(x)-f(x)\cdot g(x_0)+f(x)\cdot g(x_0)-f(x_0)\cdot g(x_0)}{x-x_0}\\ & = \displaystyle \lim_{x\to x_0}\frac{f(x)\cdot (g(x)- g(x_0))+(f(x)-f(x_0))\cdot g(x_0)}{x-x_0}\\ & = \displaystyle \lim_{x\to x_0}f(x)\cdot\frac{g(x)- g(x_0)}{x-x_0}+\lim_{x\to x_0}\frac{f(x)-f(x_0)}{x-x_0}\cdot g(x_0)=f(x_0)\cdot g'(x_0)+f'(x_0)\cdot g(x_0). \end{array} \]

La derivada del cociente de funciones

ProposiciĂ³n.

Sean \(f\) y \(g\) dos funciones reales de variable real y sea \(x_0\) un punto del dominio de \(f\) y de \(g\) tal que \(g'(x_0)\neq 0\). Si \(f\) y \(g\) son derivables en \(x_0\), tambiĂ©n lo es la funciĂ³n cociente \(\frac{f}{g}\), y se verifica que: \(\left(\frac{f}{g}\right)'(x_0)=\frac{f'(x_0)\cdot g(x_0)-f(x_0)\cdot g'(x_0)}{g(x_0)^2}\).

DemostraciĂ³n: \[ \begin{array}{rl} \left(\frac{f}{g}\right)'(x_0) & = \displaystyle\lim_{x\to x_0}\frac{\left(\frac{f}{g}\right)(x)-\left(\frac{f}{g}\right)(x_0)}{x-x_0}=\lim_{x\to x_0}\frac{\frac{f(x)}{g(x)}-\frac{f(x_0)}{g(x_0)}}{x-x_0} =\lim_{x\to x_0} \frac{f(x)\cdot g(x_0)-g(x)\cdot f(x_0)}{g(x)\cdot g(x_0)\cdot (x-x_0)} \\ & \displaystyle =\lim_{x\to x_0} \frac{f(x)\cdot g(x_0)-f(x_0)\cdot g(x_0)+f(x_0)\cdot g(x_0) -g(x)\cdot f(x_0)}{g(x)\cdot g(x_0)\cdot (x-x_0)} \\ & = \displaystyle \lim_{x\to x_0}\frac{1}{g(x)\cdot g(x_0)}\cdot \left(\lim_{x\to x_0}\frac{f(x)-f(x_0)}{x-x_0}\cdot g(x_0)-f(x_0)\cdot \lim_{x\to x_0} \frac{g(x)-g(x_0)}{x-x_0}\right) \\ & = \displaystyle \frac{1}{g(x_0)^2}(f'(x_0)\cdot g(x_0)-f(x_0)\cdot g'(x_0)). \end{array} \]

Derivada de la funciĂ³n inversa

ProposiciĂ³n.

Sea \(f\) una funciĂ³n real de variable real y \(x_0\) un punto del dominio de \(f\). Suponemos que \(f\) es derivable en \(x_0\) y que \(f'(x_0)\neq 0\). Supongamos que \(f\) admite inversa \(g\), es decir, existe una funciĂ³n \(g\) tal que \(g\circ f=\mathrm{Id}\), es decir, para cualquier valor \(x\) del dominio de \(f\), se cumple que \(g(f(x))=x\). AdemĂ¡s, suponemos que \(g\) es continua en \(f(x_0)\). Entonces \(g\) es derivable en \(f(x_0)\) y se verifica que \[g'(f(x_0))=\frac{1}{f'(x_0)}.\]

Derivada de la funciĂ³n inversa

DemostraciĂ³n:

El valor de \(g'(f(x_0))\) es: \(g'(f(x_0)) = \displaystyle\lim_{y\to f(x_0)}\frac{g(y)-g(f(x_0))}{y-f(x_0)}.\)

Para el cĂ¡lculo del lĂ­mite anterior, hacemos el cambio de variable siguiente: \(y=f(x)\), o lo que es lo mismo, \(x=g(y)\). Como \(y\) tiende a \(f(x_0)\), tendremos con la variable nueva que \(f(x)\) tiende a \(f(x_0)\) pero como \(g\) es continua en \(f(x_0)\), deducimos que \(g(f(x))=x\) tiende a \(g(f(x_0))=x_0\). En resumen, el lĂ­mite anterior puede escribirse como: \[ g'(f(x_0)) =\lim_{x\to x_0}\frac{g(f(x))-g(f(x_0))}{f(x)-f(x_0)}=\lim_{x\to x_0}\frac{x-x_0}{f(x)-f(x_0)} = \frac{1}{\displaystyle\lim_{x\to x_0}\frac{f(x)-f(x_0)}{x-x_0}}=\frac{1}{f'(x_0)}. \]

Ejemplo

Ejemplo: derivada de la funciĂ³n \(g(x)=\sqrt[n]{x}\).

La funciĂ³n \(g(x)=\sqrt[n]{x}\) es la inversa de la funciĂ³n \(f(x)=x^n\) ya que: \(g(f(x))=g(x^n)=\sqrt[n]{x^n}=x\).

Usando la expresiĂ³n vista anteriormente, podemos escribir que: \(g'(f(x))=g'(x^n)=\frac{1}{f'(x)}\).

En un ejemplo anterior vimos que \(f'(x)=n\cdot x^{n-1}\). Por tanto: \(g'(x^n)=\frac{1}{n\cdot x^{n-1}}\).

Sea \(y=x^n\), entonces \(x=\sqrt[n]{y}\). Por tanto, \(g'(y)=\frac{1}{n\cdot x^{n-1}}=\frac{1}{n\cdot \sqrt[n]{y^{n-1}}}\).

La derivada de la funciĂ³n \(g(x)\) en Wolfram Alpha se muestra en el enlace siguiente:

Ejemplo

Ejemplo: derivada de la funciĂ³n \(\tan x\).

La funciĂ³n \(\tan x\) estĂ¡ definida como \(\frac{\sin x}{\cos x}\).

Calculemos primero la derivada de la funciĂ³n \(\sin x\) en un valor \(x_0\): \[ \begin{array}{rl} (\sin)'(x_0) & \displaystyle =\lim_{x\to x_0}\frac{\sin x-\sin x_0}{x-x_0}=\lim_{x\to x_0}\frac{2\cos\left(\frac{x+x_0}{2}\right)\cdot\sin\left(\frac{x-x_0}{2}\right)}{x-x_0}=2\cdot \cos(x_0)\lim_{x\to x_0}\frac{\sin\left(\frac{x-x_0}{2}\right)}{x-x_0} \\ & \displaystyle = \cos(x_0)\lim_{x\to x_0}\frac{\sin\left(\frac{x-x_0}{2}\right)}{\frac{x-x_0}{2}} = \cos(x_0), \end{array} \] usando que \(\lim\limits_{x\to x_0}\frac{\sin\left(\frac{x-x_0}{2}\right)}{\frac{x-x_0}{2}}=1.\) La expresiĂ³n anterior se deduce del hecho de que \(\lim\limits_{t\to 0} \frac{\sin t}{t}=1\).

Si se hace el cambio de variable \(t=\frac{x-x_0}{2}\) y teniendo en cuenta que como \(x\to x_0\), entonces \(t\to 0\), los dos lĂ­mites anteriores son iguales a 1.

Ejemplo

Ejemplo: derivada de la funciĂ³n \(\tan x\).

Para calcular la derivada de la funciĂ³n \(\cos x\), usamos una tĂ©cnica similar: \[ \begin{array}{rl} (\cos)'(x_0) & \displaystyle =\lim_{x\to x_0}\frac{\cos x-\cos x_0}{x-x_0}=\lim_{x\to x_0}\frac{-2\sin\left(\frac{x+x_0}{2}\right)\cdot\sin\left(\frac{x-x_0}{2}\right)}{x-x_0}=-2\cdot \sin(x_0)\lim_{x\to x_0}\frac{\sin\left(\frac{x-x_0}{2}\right)}{x-x_0} \\ & \displaystyle =- \sin(x_0)\lim_{x\to x_0}\frac{\sin\left(\frac{x-x_0}{2}\right)}{\frac{x-x_0}{2}} = -\sin(x_0). \end{array} \] Usamos la propiedad de la derivada del cociente, podemos hallar la derivada de la funciĂ³n \(\tan x\): \[ \begin{array}{rl} (\tan)'(x_0) & =\left(\frac{\sin}{\cos}\right)'(x_0)=\frac{\sin'(x_0)\cdot \cos(x_0)-\sin (x_0)\cdot \cos'(x_0)}{\cos^2(x_0)}=\frac{\cos(x_0)\cdot \cos(x_0)+\sin (x_0)\cdot \sin(x_0)}{\cos^2(x_0)} \\ & = \frac{\cos^2(x_0)+\sin^2(x_0)}{\cos^2(x_0)}=\frac{1}{\cos^2 (x_0)}=1+\tan^2 (x_0). \end{array} \]

La derivada de la funciĂ³n \(\tan x\) en Wolfram Alpha se muestra en el enlace siguiente:

Derivada de la funciĂ³n compuesta. Regla de la cadena

ProposiciĂ³n.

Sean \(f\) y \(g\) dos funciones reales de variable real. Sea \(x_0\) un punto del dominio de \(f\) tal que \(f(x_0)\) es del dominio de \(g\). Supongamos que \(g\) es derivable en \(f(x_0)\) y \(f\) es derivable en \(x_0\). Entonces la funciĂ³n compuesta \(g\circ f\) es derivable en \(x_0\) y se verifica que:

\[(g\circ f)'(x_0)=g'(f(x_0))\cdot f'(x_0).\]

Derivada de la funciĂ³n compuesta. Regla de la cadena

DemostraciĂ³n: \[ \begin{array}{rl} (g\circ f)'(x_0) & \displaystyle =\lim_{x\to x_0}\frac{(g\circ f)(x)-(g\circ f)(x_0)}{x-x_0}=\lim_{x\to x_0}\frac{g(f(x))-g(f(x_0))}{x-x_0} \\ & \displaystyle =\lim_{x\to x_0}\frac{g(f(x))-g(f(x_0))}{f(x)-f(x_0)}\cdot \lim_{x\to x_0}\frac{f(x)-f(x_0)}{x-x_0}=g'(f(x_0))\cdot f'(x_0). \end{array} \] En el Ăºltimo cĂ¡lculo, podemos afirmar que el lĂ­mite \(\displaystyle\lim_{x\to x_0}\frac{g(f(x))-g(f(x_0))}{f(x)-f(x_0)}\) vale \(g'(f(x_0))\) porque como \(f\) es derivable en \(x_0\), \(f\) serĂ¡ continua en \(x_0\) y si \(x\to x_0\), entonces \(f(x)\to f(x_0)\) y el lĂ­mite anterior queda: \[ \lim_{x\to x_0}\frac{g(f(x))-g(f(x_0))}{f(x)-f(x_0)} =\lim_{f(x)\to f(x_0)}\frac{g(f(x))-g(f(x_0))}{f(x)-f(x_0)}=g'(f(x_0)). \]

Ejemplo

Ejemplo

Hallemos la derivada de la funciĂ³n \(h(x)=\arctan (x^3+x^2+1)\).

La funciĂ³n anterior es la composiciĂ³n de la funciĂ³n \(f(x)=x^3+x^2+1\) y \(g(y)=\arctan y\), es decir: \(h(x)=(g\circ f)(x)\).

La derivada de la funciĂ³n \(f(x)\) se puede calcular usando la propiedad de la suma de derivadas y la expresiĂ³n de la derivada del monomio \(x^n\): \[ (x^3+x^2+1)'=(x^3)'+(x^2)'+(1)' = 3x^2+2x+0=3x^2+2x. \]

A continuaciĂ³n, hallemos la derivada de la funciĂ³n \(g(y)=\arctan y\). Dicha funciĂ³n es la funciĂ³n inversa de la funciĂ³n \(\tan x\). Usando la expresiĂ³n de la derivada de la funciĂ³n inversa, tenemos: \[ g'(\tan x)=\frac{1}{\tan'(x)}=\frac{1}{\frac{1}{\cos^2 x}}=\cos^2 x. \]

Ejemplo

Ejemplo

Para hallar \(g'(y)\), tenemos que escribir \(y=\tan x\), y, por tanto, \(x=\arctan y\): \[ g'(y)=\cos^2(\arctan y)=\frac{1}{1+\tan^2 (\arctan y)}=\frac{1}{1+y^2}, \] donde hemos usado la siguiente relaciĂ³n trigonomĂ©trica: \(\cos^2\alpha = \frac{1}{1+\tan^2\alpha}\).

La derivada de la funciĂ³n \(h(x)\) usando la regla de la cadena serĂ¡: \[ \begin{array}{rl} h'(x) & =(g\circ f)'(x)=g'(f(x))\cdot f'(x)=\frac{1}{1+f(x)^2}\cdot (3 x^2+2x)=\frac{3x^2+2x}{1+(x^3+x^2+1)^2}\\ & =\frac{3x^2+2x}{x^6+2 x^5+x^4+2 x^3+2 x^2+2}. \end{array} \]

La derivada de la funciĂ³n \(h(x)\) en Wolfram Alpha se muestra en el enlace siguiente:

Tablas de derivadas

Si vais al enlace siguiente tablas de derivadas y escribĂ­s “tablas de derivadas” en la casilla de bĂºsqueda, encontrarĂ©is un montĂ³n de tablas de derivadas para las funciones mĂ¡s usadas.

Teoremas de derivaciĂ³n

DerivaciĂ³n y extremos

DefiniciĂ³n de mĂ¡ximo relativo de una funciĂ³n.

Sea \(f: (a,b)\longrightarrow \mathbb{R}\) una funciĂ³n real de variable real. Sea \(x_0\in (a,b)\) un valor del dominio de \(f\). Diremos que \(f\) tiene un mĂ¡ximo relativo en el punto \(x_0\) si existe un entorno de \(x_0\), es decir, existe un valor \(\delta >0\), tal que para todo valor \(x\) de este entorno, es decir, \(x\in (x_0-\delta,x_0+\delta)\subseteq (a,b)\), se cumple que \(f(x)\leq f(x_0)\).

Si la condiciĂ³n anterior se verifica para cualquier punto \(x\in (a,b)\), diremos que \(f\) tiene un mĂ¡ximo absoluto en el punto \(x_0\).

DerivaciĂ³n y extremos

DefiniciĂ³n de mĂ­nimo relativo de una funciĂ³n.

Sea \(f: (a,b)\longrightarrow \mathbb{R}\) una funciĂ³n real de variable real. Sea \(x_0\in (a,b)\) un valor del dominio de \(f\). Diremos que \(f\) tiene un mĂ­nimo relativo en el punto \(x_0\) si existe un entorno de \(x_0\), es decir, existe un valor \(\delta >0\), tal que para todo punto \(x\) de este entorno, es decir, \(x\in (x_0-\delta,x_0+\delta)\subseteq (a,b)\), se cumple que \(f(x)\geq f(x_0)\).

Si la condiciĂ³n anterior se verifica para cualquier valor \(x\in (a,b)\), diremos que \(f\) tiene un mĂ­nimo absoluto en el punto \(x_0\).

Los puntos donde \(f\) presente un mĂ¡ximo o un mĂ­nimo relativos o absolutos se denominan extremos relativos o absolutos de la funciĂ³n.

Ejemplo

Ejemplo

Consideremos la funciĂ³n siguiente definida en el intervalo \((-2\pi,2\pi)\): \[ \begin{array}{rl} f:(-2\pi,2\pi) & \longrightarrow \mathbb{R},\\ x& \longrightarrow \sin(x)-\cos(x). \end{array} \] El grĂ¡fico de dicha funciĂ³n puede observase en la figura siguiente.

Vemos que tiene dos mĂ¡ximos relativos en los valores \(x=-\frac{5}{4}\pi\) y \(x=\frac{3}{4}\pi\) y dos mĂ­nimos relativos en los valores \(x=-\frac{\pi}{4}\) y \(x=\frac{7}{4}\pi\).

Observamos que dichos mĂ¡ximos y mĂ­nimos son absolutos.

El grĂ¡fico de la funciĂ³n anterior puede verse en Wolfram Alpha en el enlace siguiente:

Ejemplo

DerivaciĂ³n y extremos

Teorema.

Sea \(f: (a,b)\longrightarrow \mathbb{R}\) una funciĂ³n real de variable real. Sea \(x_0\in (a,b)\) un valor del dominio de \(f\) que sea extremo relativo de la funciĂ³n. Entonces si \(f\) es derivable en el punto \(x_0\), se verifica que \(f'(x_0)=0\).

Intuitivamente, el teorema anterior nos dice que si la funciĂ³n \(f\) tiene un comportamiento suave en el extremo \(x_0\), la recta tangente en este punto tiene que ser horizontal, es decir, su pendiente tiene que ser nula, tal como podemos observar en los extremos del ejemplo anterior.

DerivaciĂ³n y extremos

DemostraciĂ³n

Como \(f\) es derivable en el punto \(x_0\), sabemos que existe el lĂ­mite siguiente \(\displaystyle\lim_{x\to x_0}\frac{f(x)-f(x_0)}{x-x_0}\).

Decir que \(f\) es derivable en \(x_0\) es equivalente a decir que la funciĂ³n siguiente: \[ g(x)=\begin{cases} \frac{f(x)-f(x_0)}{x-x_0}, & \mbox{si }x\neq x_0,\\ f'(x_0), &\mbox{si }x=x_0, \end{cases} \] es continua en \(x_0\).

Veamos a continuaciĂ³n que necesariamente \(g(x_0)=f'(x_0)=0\) y quedarĂ¡ demostrado el teorema.

Supongamos que \(g(x_0)>0\). Como la funciĂ³n \(g\) es continua en \(x_0\), por el teorema de conservaciĂ³n del signo de funciones continuas, existirĂ¡ un entorno de \(x_0\), es decir, existirĂ¡ un \(\delta >0\) tal que si \(x\in (x_0-\delta,x_0+\delta)\subseteq (a,b)\), \(g(x)>0\) tambiĂ©n serĂ¡ positiva para todos los valores \(x\) de dicho entorno.

DerivaciĂ³n y extremos

DemostraciĂ³n

Es decir, para todo \(x\in (x_0-\delta,x_0+\delta)\), \(g(x)=\frac{f(x)-f(x_0)}{x-x_0}>0\).

Esto es equivalente a decir que el numerador y el denominador de la fracciĂ³n anterior tienen el mismo signo, dicho en otras palabras:

  • si \(x\in (x_0-\delta,x_0+\delta)\) y \(x>x_0\), entonces \(f(x) > f(x_0)\) y,
  • si \(x\in (x_0-\delta,x_0+\delta)\) y \(x<x_0\), entonces \(f(x) < f(x_0)\).

Las condiciones anteriores contradicen el hecho que \(x_0\) sea un extremo ya que hemos encontrado valores en un entorno del mismo que superan \(f(x_0)\) y valores del mismo entorno que son menores que \(f(x_0)\). En conclusiĂ³n, llegamos a una contradicciĂ³n al suponer que \(g(x_0)=f'(x_0)>0\).

Ejercicio

Suponer que \(g(x_0)=f'(x_0)<0\) y ver que se llega a una contradicciĂ³n razonando de manera similar.

DerivaciĂ³n y extremos

DemostraciĂ³n

Al no poder ser que \(f'(x_0)>\) ni \(f'(x_0)<0\), necesariamente \(f'(x_0)=0\) tal como querĂ­amos demostrar.

ObservaciĂ³n.

El recĂ­proco del teorema anterior es falso. Es decir, el hecho que \(f'(x_0)\) sea \(0\), no implica que \(x_0\) sea un extremo relativo de la funciĂ³n.

Ejemplo

Considerar, por ejemplo, la funciĂ³n \(f(x)=x^2\cdot \sin(x)\). Si derivamos dicha funciĂ³n, obtenemos: \[ f'(x)=2x\sin(x)+x^2\cos(x). \] Vemos que \(f'(0)=0\). En cambio \(f\) no tiene ningĂºn extremo en este punto tal como se observa en el grĂ¡fico de su funciĂ³n:

DerivaciĂ³n y extremos

Ejemplo

DerivaciĂ³n y extremos

Ejemplo

Para ver que el 0 no es extremo relativo, es facil ver que para \(x\in (0,\pi)\), \(f(x)\geq 0\) y para \(x\in (-\pi,0)\), \(f(x)\leq 0\). Comprobémoslo para unos cuantos valores en python, para \(x=-0.2,-0.1,0.1,0.2\):

from sympy import * 
  
def f(x):
 return(x**2*sin(x))

DerivaciĂ³n y extremos

Ejemplo

for x in [-0.2,-0.1,0.1,0.2]:
  print('f({x})={res}'.format(x=x, res=f(x)))
## f(-0.2)=-0.00794677323180245
## f(-0.1)=-0.000998334166468282
## f(0.1)=0.000998334166468282
## f(0.2)=0.00794677323180245

DerivaciĂ³n y extremos

Ejemplo

El cĂ¡lculo de la derivada de la funciĂ³n anterior puede verse en Wolfram Alpha en el enlace siguiente:

El grĂ¡fico de la funciĂ³n anterior puede verse en Wolfram Alpha en el enlace siguiente:

DerivaciĂ³n y extremos

DefiniciĂ³n de crecimiento de una funciĂ³n.

Sea \(f: (a,b)\longrightarrow \mathbb{R}\) una funciĂ³n real de variable real. Sea \(x_0\in (a,b)\) un valor del dominio de \(f\). Diremos que \(f\) es estrictamente creciente en el punto \(x_0\) si existe un entorno de \(x_0\), es decir, existe un valor \(\delta >0\) tal que para todo \(x\) de dicho entorno diferente de \(x_0\), o si \(x\in (x_0-\delta,x_0+\delta)\subseteq (a,b)\), con \(x\neq x_0\), se verifica que el cociente siguiente es positivo: \(\frac{f(x)-f(x_0)}{x-x_0}>0\).

ObservaciĂ³n.

La definiciĂ³n anterior es equivalente a decir que existe un valor \(\delta >0\) tal que si \(x\in (x_0,x_0+\delta)\) o \(x>x_0\), entonces \(f(x)>f(x_0)\) y si \(x\in (x_0-\delta, x_0)\) o \(x<x_0\), entonces \(f(x)<f(x_0)\). Ver el grĂ¡fico siguiente.

DerivaciĂ³n y extremos

DerivaciĂ³n y extremos

DefiniciĂ³n de decrecimiento de una funciĂ³n.

Sea \(f: (a,b)\longrightarrow \mathbb{R}\) una funciĂ³n real de variable real. Sea \(x_0\in (a,b)\) un valor del dominio de \(f\). Diremos que \(f\) es estrictamente decreciente en el punto \(x_0\) si existe un entorno de \(x_0\), es decir, existe un valor \(\delta >0\) tal que para todo \(x\) de dicho entorno diferente de \(x_0\), o si \(x\in (x_0-\delta,x_0+\delta)\subseteq (a,b)\), con \(x\neq x_0\), se verifica que el cociente siguiente es negativo: \(\frac{f(x)-f(x_0)}{x-x_0}<0\).

ObservaciĂ³n.

La definiciĂ³n anterior es equivalente a decir que existe un valor \(\delta >0\) tal que si \(x\in (x_0,x_0+\delta)\) o \(x>x_0\), entonces \(f(x)<f(x_0)\) y si \(x\in (x_0-\delta, x_0)\) o \(x<x_0\), entonces \(f(x)>f(x_0)\). Ver grĂ¡fico siguiente.

DerivaciĂ³n y extremos

DerivaciĂ³n y extremos

ObservaciĂ³n.

Si la desigualdad \(\frac{f(x)-f(x_0)}{x-x_0}\geq 0\) no es estricta, se dice que \(f\) es creciente.

ObservaciĂ³n.

Si la desigualdad \(\frac{f(x)-f(x_0)}{x-x_0}\leq 0\) no es estricta, se dice que \(f\) es decreciente.

DerivaciĂ³n y extremos

ProposiciĂ³n.

Sea \(f: (a,b)\longrightarrow \mathbb{R}\) una funciĂ³n real de variable real. Sea \(x_0\in (a,b)\) un valor del dominio de \(f\). Supongamos que \(f\) es derivable en \(x_0\). Entonces si \(f'(x_0)>0\) o \(f\) tiene derivada positiva en \(x_0\), entonces \(f\) es estrictamente creciente en \(x_0\).

DemostraciĂ³n

Como \(\displaystyle f'(x_0)=\lim_{x\to x_0}\frac{f(x)-f(x_0)}{x-x_0}>0\), usando la definiciĂ³n de lĂ­mite tenemos que: \[ \forall \epsilon >0,\ \exists\delta >0\ \mbox{t.q. si }|x-x_0|<\delta,\mbox{ entonces }\left|f'(x_0)-\frac{f(x)-f(x_0)}{x-x_0}\right|< \epsilon. \]

DerivaciĂ³n y extremos

DemostraciĂ³n

La Ăºltima condiciĂ³n se puede escribir de la siguiente manera: \[ f'(x_0)-\epsilon < \frac{f(x)-f(x_0)}{x-x_0} < f'(x_0)+\epsilon. \] Como \(f'(x_0)>0\), siempre es posible hallar un \(\epsilon >0\) tal que \(f'(x_0)-\epsilon>0\).

Para este \(\epsilon >0\), podemos encontrar un entorno de \(x_0\), es decir, un \(\delta >0\) tal que si \(x\in (x_0-\delta,x_0+\delta)\), entonces: \[ 0<f'(x_0)-\epsilon < \frac{f(x)-f(x_0)}{x-x_0} < f'(x_0)+\epsilon, \] de donde deducimos que \(\frac{f(x)-f(x_0)}{x-x_0}>0\) para todo \(x\) del entorno, \(x\in (x_0-\delta,x_0+\delta)\), lo que equivale a decir que \(f\) es estrictamente creciente en \(x_0\).

DerivaciĂ³n y extremos

ProposiciĂ³n.

Sea \(f: (a,b)\longrightarrow \mathbb{R}\) una funciĂ³n real de variable real. Sea \(x_0\in (a,b)\) un valor del dominio de \(f\). Supongamos que \(f\) es derivable en \(x_0\). Entonces si \(f'(x_0)<0\) o \(f\) tiene derivada negativa en \(x_0\), entonces \(f\) es estrictamente decreciente en \(x_0\).

Ejercicio

Demostrar la proposiciĂ³n anterior usando la misma tĂ©cnica que para demostrar que si \(f'(x_0)>0\), entonces \(f\) es estrictamente creciente en \(x_0\).

Ejemplo

Ejemplo

Consideremos la funciĂ³n siguiente de un ejemplo anterior: \(f(x)=\sin(x)-\cos(x)\).

La derivada de la funciĂ³n \(f\) vale: \(f'(x)=\cos(x)+\sin(x)\).

Dicha funciĂ³n se anula en los extremos \(x=-\frac{5}{4}\pi,-\frac{\pi}{4},\frac{3}{4}\pi\) y \(\frac{7}{4}\pi\):

def df(x):
 return(cos(x)+sin(x))
 
for x in [-5*pi/4,-pi/4,3*pi/4,7*pi/4]:
  print("f'({x})={res}".format(x=x, res=df(x)))
## f'(-5*pi/4)=0
## f'(-pi/4)=0
## f'(3*pi/4)=0
## f'(7*pi/4)=0

Ejemplo

En los puntos \(x=-\frac{3}{2}\pi\) y \(x=\frac{\pi}{2}\) la funciĂ³n es estrictamente creciente al tener derivada positiva en dichos puntos:

for x in [-3*pi/2,pi/2]:
  print("f'({x})={res}".format(x=x, res=df(x)))
## f'(-3*pi/2)=1
## f'(pi/2)=1

En cambio, en los puntos \(x=-\frac{\pi}{2}\) y \(\frac{3}{2}\pi\) la funciĂ³n es estrictamente decreciente al tener derivada negativa en dichos puntos:

for x in [-pi/2,3*pi/2]:
  print("f'({x})={res}".format(x=x, res=df(x)))
## f'(-pi/2)=-1
## f'(3*pi/2)=-1

Teoremas de Rolle y del valor medio

Las proposiciones vistas hasta ahora nos permiten determinar el comportamiento local de una funciĂ³n en un punto en tĂ©rminos de su crecimiento dependiendo del signo de la derivada de dicha funciĂ³n en dicho punto.

Vamos a ver dos resultados que nos permiten determinar las propiedades globales de la funciĂ³n en todo su dominio a partir del comportamiento de la funciĂ³n derivada de dicha funciĂ³n.

El teorema de Rolle dice que si los valores de una funciĂ³n derivable en todo el interior de un intervalo, los valores coinciden en los extremos del mismo, necesariamente ha de tener al menos un mĂ¡ximo o un mĂ­nimo Intuitivamente, el resultado es claro ya que si suponemos por ejemplo que la funciĂ³n crece en su extremo izquierdo \(a\), como \(f(a)=f(b)\), donde \(b\) es su extremo derecho, en algĂºn momento tiene que decrecer. Por tanto, en dicho momento, la funciĂ³n tendrĂ¡ un extremo o un mĂ¡ximo en este caso.

Teoremas de Rolle y del valor medio

Teoremas de Rolle y del valor medio

El teorema del valor medio dice que dada una funciĂ³n derivable en todo su dominio, ha de existir un punto en el que recta tangente en dicho punto sea paralela a la recta que pasa por los extremos del dominio de la funciĂ³n.

En el grĂ¡fico siguiente la recta verde es la recta que pasa por los extremos de la funciĂ³n (en azul) y la recta roja es la recta tangente al punto \(c\) que pertenece al dominio de la funciĂ³n.

Teoremas de Rolle y del valor medio

Teorema de Rolle

Teorema de Rolle.

Sea \(f: [a,b]\longrightarrow \mathbb{R}\) una funciĂ³n real de variable real. Supongamos que \(f\) es continua en todo su dominio \([a,b]\) y derivable en los puntos del interior \((a,b)\). Supongamos ademĂ¡s que las imĂ¡genes en los extremos coinciden, es decir, \(f(a)=f(b)\). Entonces existe al menos un punto \(c\in (a,b)\) tal que \(f'(c)=0\).

ObservaciĂ³n.

En el dominio de definiciĂ³n de \(f\) estĂ¡n incluidos los extremos del intervalo.

Teorema de Rolle

DemostraciĂ³n

Como la funciĂ³n \(f\) es continua en el intervalo cerrado \([a,b]\), deducimos que \(f\) tiene un mĂ¡ximo absoluto \(M\) y un mĂ­nimo absoluto \(m\).

Pueden ocurrir dos casos:

  • Que el mĂ¡ximo absoluto se alcance en el extremo \(a\) y el mĂ­nimo, en el extremo \(b\), con \(f(a)=M\) y \(f(b)=m\) o al revĂ©s, es decir, que el mĂ¡ximo absoluto se alcance en el extremo \(b\) y el mĂ­nimo, en el extremo \(a\), con \(f(a)=m\) y \(f(b)=M\). Como \(f(a)=f(b)\), resulta que \(M=m\) y la Ăºnica funciĂ³n en un intervalo en donde su mĂ¡ximo coincide con su mĂ­nimo es la funciĂ³n constante. En este caso \(f'(x)=0\), para todo \(x\in (a,b)\) como ya vimos anteriormente y el teorema quedarĂ­a demostrado en este caso.
  • Supongamos que el mĂ¡ximo o el mĂ­nimo de la funciĂ³n se alcanza en un punto \(c\in (a,b)\) del interior del intervalo. Por un teorema visto anteriormente, tenemos que \(f'(c)=0\) y el teorema quedarĂ­a demostrado tambiĂ©n en este caso.

Teorema del valor medio de Cauchy

Teorema del valor medio de Cauchy.

Sean \(f,g:[a,b]\longrightarrow \mathbb{R}\) dos funciones continuas en \([a,b]\) y derivables en \((a,b)\). Entonces, existe un punto \(c\in (a,b)\) del interior del intervalo tal que: \[ f'(c)\cdot (g(b)-g(a)) = g'(c)\cdot (f(b)-f(a)). \]

Teorema del valor medio de Cauchy

DemostraciĂ³n

Consideramos la funciĂ³n siguiente: \[ h(x)=f(x)\cdot (g(b)-g(a))-g(x)\cdot (f(b)-f(a)), \] que serĂ¡ continua en \([a,b]\) y derivable en \((a,b)\) al ser suma de productos de funciones continuas y derivables por constantes.

La idea es aplicar el teorema de Rolle a la funciĂ³n \(h(x)\). Calculemos \(h(a)\) y \(h(b)\): \[ \begin{array}{rl} h(a) & = f(a)\cdot (g(b)-g(a))-g(a)\cdot (f(b)-f(a)) = f(a)\cdot g(b)-g(a)\cdot f(b),\\ h(b) & = f(b)\cdot (g(b)-g(a))-g(b)\cdot (f(b)-f(a)) = -f(b)\cdot g(a)+g(b)\cdot f(a). \end{array} \] Se cumple, por tanto, que \(h(a)=h(b)\). Aplicando el teorema de Rolle a la funciĂ³n \(h\), tenemos que existe un punto \(c\in (a,b)\) tal que \(h'(c)=0\): \[ h'(c)=f'(c)\cdot (g(b)-g(a))-g'(c)\cdot (f(b)-f(a))=0, \] de donde deducimos lo que dice la tesis del teorema: \[ f'(c)\cdot (g(b)-g(a)) = g'(c)\cdot (f(b)-f(a)). \]

Teorema del valor medio de Lagrange

Corolario: Teorema del valor medio de Lagrange.

Sea \(f:[a,b]\longrightarrow \mathbb{R}\) una funciĂ³n continua en \([a,b]\) y derivable en \((a,b)\). Entonces, existe un punto \(c\in (a,b)\) del interior del intervalo tal que: \[ f(b)-f(a)=f'(c)\cdot (b-a). \]

Ejercicio

Demostrar el teorema del valor medio de Lagrange.

IndicaciĂ³n: considerar \(f(x)\) la funciĂ³n del teorema, \(g(x)=x\) y aplicar el teorema del valor medio de Cauchy.

Teorema del valor medio de Lagrange

ObservaciĂ³n.

El Teorema del valor medio de Lagrange es equivalente a afirmar lo que hemos dicho anteriormente: si \(f\) es derivable en el intervalo abierto y continua en el cerrado, existe un punto \(c\) del interior del intervalo tal que \(f'(c)=\frac{f(b)-f(a)}{b-a}\), es decir, la pendiente de la recta tangente en el punto \(c\) coincide con la pendiente de la recta que pasa por los extremos \((a,f(a))\) y \((b,f(b))\).

Es decir, la recta tangente en el punto \(c\) es paralela a la recta que pasa por los extremos del intervalo de definiciĂ³n de la funciĂ³n \(f\).

Consecuencias de los teoremas

Corolario.

Sea \(f:[a,b]\longrightarrow \mathbb{R}\) una funciĂ³n continua en \([a,b]\) y derivable en \((a,b)\) tal que \(f'(x)=0\) para todo \(x\in (a,b)\). Entonces \(f\) es constante.

DemostraciĂ³n

Sea \(x\in (a,b]\). Veamos que \(f(x)=f(a)\) y, por tanto, \(f\) serĂ¡ constante.

Para ello, consideremos la funciĂ³n \(f\) restringida al intervalo \([a,x]\), \(f:[a,x]\longrightarrow\mathbb{R}\), que serĂ¡ continua en \([a,x]\) y derivable en \((a,x)\). Si aplicamos el teorema del valor medio de Lagrange, tenemos que existe un \(c\in (a,x)\) tal que: \[ f(x)-f(a)=f'(c)\cdot (x-a)=0, \] ya que nos dicen que \(f'(c)=0\) al ser la derivada nula en cualquier punto del intervalo \((a,b)\).

Deducimos, por tanto, que \(f(x)=f(a)\), condiciĂ³n que equivale a que la funciĂ³n \(f\) es constante.

Consecuencias de los teoremas

Corolario.

Sean \(f, g:[a,b]\longrightarrow \mathbb{R}\) funciones continuas en \([a,b]\) y derivables en \((a,b)\) tal que \(f'(x)=g'(x)\) para todo \(x\in (a,b)\). Entonces \(f(x)-g(x)\) es constante.

Ejercicio

Demostrar el corolario anterior.

IndicaciĂ³n: aplicar el resultado que hemos visto antes a la funciĂ³n \(h(x)=f(x)-g(x)\).

Consecuencias de los teoremas

Corolario.

Sea \(f:(a,b)\longrightarrow \mathbb{R}\) una funciĂ³n derivable en todo punto del intervalo \((a,b)\). Entonces, \(f\) es creciente en \((a,b)\) si, y sĂ³lo si, \(f'(x)\geq 0\), para todo \(x\in (a,b)\) del intervalo.

DemostraciĂ³n

\(\Rightarrow\) Supongamos que la funciĂ³n \(f\) es creciente en \((a,b)\). Esto significa que fijado \(x_0\in (a,b)\) en el intervalo, existe un entorno de \(x_0\), es decir, existe un \(\delta >0\), tal que para todo valor \(x\in (x_0-\delta,x_0+\delta)\), se verifica que \(\frac{f(x)-f(x_0)}{x-x_0}\geq 0\).

Entonces, usando que \(\displaystyle f'(x_0)=\lim_{x\to x_0} \frac{f(x)-f(x_0)}{x-x_0}\), tendremos que \(f'(x_0)\geq 0\), tal como querĂ­amos ver.

Consecuencias de los teoremas

DemostraciĂ³n

\(\Leftarrow\) Supongamos ahora que \(f'(x_0)\geq 0\), para todo valor \(x_0\in (a,b)\) dentro del intervalo. Veamos que \(f\) es creciente en \(x_0\).

Sea \(x>x_0\). Si aplicamos el teorema del valor medio de Lagrange a la funciĂ³n \(f\) restringida al intervalo \([x_0,x]\) tenemos que existe un punto \(c\) tal que: \[ f'(c)=\frac{f(x)-f(x_0)}{x-x_0}\geq 0. \] Sea ahora \(x<x_0\). Si volvemos a aplicar el el teorema del valor medio de Lagrange a la funciĂ³n \(f\) restringida al intervalo \([x,x_0]\) tenemos que existe un punto \(c\) tal que: \[ f'(c)=\frac{f(x_0)-f(x)}{x_0-x}=\frac{f(x)-f(x_0)}{x-x_0}\geq 0. \]

Consecuencias de los teoremas

DemostraciĂ³n

En resumen, el cociente \(\frac{f(x)-f(x_0)}{x-x_0}\geq 0\) siempre es positivo, condiciĂ³n que equivale a afirmar que \(f\) es creciente en \(x_0\).

De hecho, hubiera sido suficiente demostrar que el cociente anterior es positivo en un entorno de \(x_0\) pero hemos demostrado mĂ¡s, hemos visto que dicho cociente siempre es positivo sea cual sea el valor \(x\in (a,b)\) del intervalo.

Consecuencias de los teoremas

Corolario.

Sea \(f:(a,b)\longrightarrow \mathbb{R}\) una funciĂ³n derivable en todo punto del intervalo \((a,b)\). Entonces, \(f\) es decreciente en \((a,b)\) si, y sĂ³lo si, \(f'(x)\leq 0\), para todo \(x\in (a,b)\) del intervalo.

Ejercicio

Demostrar el corolario anterior usando la misma tĂ©cnica de demostraciĂ³n para el caso en que la funciĂ³n \(f\) es creciente.

Ejemplo

Ejemplo

Consideremos la funciĂ³n vista anteriormente \(f(x)=\sin (x)-\cos(x)\) definida en el intervalo \((-\pi,\pi)\).

Como \(f(-\pi)=1=f(\pi)\), aplicando el teorema de Rolle, sabemos que existe como mĂ­nimo un punto \(c\) tal que \(f'(c)=0\). De hecho, hay dos como hemos visto anteriormente: \(c=-\frac{\pi}{4}\) y \(c=\frac{3}{4}\pi\): \[ \begin{array}{rl} f'(x) & =\cos(x)+\sin(x),\\ f'\left(-\frac{\pi}{4}\right) & =\cos\left(-\frac{\pi}{4}\right)+\sin\left(-\frac{\pi}{4}\right)=\frac{\sqrt{2}}{2}-\frac{\sqrt{2}}{2}=0,\\ f'\left(\frac{3\pi}{4}\right) & =\cos\left(\frac{3\pi}{4}\right)+\sin\left(\frac{3\pi}{4}\right)=-\frac{\sqrt{2}}{2}+\frac{\sqrt{2}}{2}=0. \end{array} \] Si ahora consideramos la funciĂ³n anterior pero definida en el intervalo \(\left(-\frac{\pi}{2},\frac{\pi}{2}\right)\), tenemos que existe un punto \(c\) tal que: \[ f'(c)=\cos(c)+\sin(c)=\frac{f\left(\frac{\pi}{2}\right)-f\left(-\frac{\pi}{2}\right)}{\frac{\pi}{2}-\left(-\frac{\pi}{2}\right)}=\frac{1-(-1)}{\pi}=\frac{2}{\pi}. \]

Ejemplo

Hallemos a continuaciĂ³n el valor \(c\): \[ \begin{array}{rl} \cos(c)+\sin(c) & = \frac{2}{\pi}, \\ \cos(c) & = \frac{2}{\pi}-\sin(c), \\ \pm \sqrt{1-\sin^2(c)} & = \frac{2}{\pi}-\sin(c), \\ 1-\sin^2(c) & = \left(\frac{2}{\pi}-\sin(c)\right)^2 = \frac{4}{\pi^2}+\sin^2(c)-\frac{4}{\pi}\sin(c),\\ 2\sin^2(c)-\frac{4}{\pi}\sin(c)+\frac{4}{\pi^2}-1 & = 0,\\ \sin(c) & = \frac{\frac{4}{\pi}\pm \sqrt{\frac{16}{\pi^2}-8\cdot\left(\frac{4}{\pi^2}-1\right)}}{4},\\ \sin(c) & = \frac{\frac{4}{\pi}\pm \sqrt{8-\frac{16}{\pi^2}}}{4}=\frac{4\pm\sqrt{8\pi^2-16}}{4\pi}=\frac{2\pm\sqrt{2\pi^2-4}}{2\pi}. \end{array} \]

Ejemplo

El valor de \(\cos(c)\) serĂ¡: \[ \begin{array}{rl} \cos(c) & =\pm\sqrt{1-\left(\frac{2\pm\sqrt{2\pi^2-4}}{2\pi}\right)^2} = \pm\sqrt{1-\frac{2\pi^2\pm 4\sqrt{2\pi^2-4}}{4\pi^2}}=\pm\sqrt{\frac{2\pi^2\mp4\sqrt{2\pi^2-4}}{4\pi^2}}\\ & =\pm\sqrt{\frac{(2\mp\sqrt{2\pi^2-4})^2}{4\pi^2}} = \pm\frac{(2\mp\sqrt{2\pi^2-4})}{2\pi}. \end{array} \] Entonces las parejas \((\sin(c),\cos(c))\) son las siguientes:

  • si \(\sin(c)=\frac{2+\sqrt{2\pi^2-4}}{2\pi}\), entonces \(\cos(c)=\frac{2-\sqrt{2\pi^2-4}}{2\pi}\),
  • si \(\sin(c)=\frac{2-\sqrt{2\pi^2-4}}{2\pi}\), entonces \(\cos(c)=\frac{2+\sqrt{2\pi^2-4}}{2\pi}\).

El primer caso no puede ser ya que \(\cos(c)<0\) y como estamos en el intervalo \(\left(-\frac{\pi}{2},\frac{\pi}{2}\right)\), la funciĂ³n coseno es positiva.

SĂ³lo serĂ¡ soluciĂ³n el segundo caso, donde el valor de \(c\) serĂ¡ aproximadamente: -0.318.

Ejemplo

Comprobemos usando python que el valor de \(c\) hallado es el correcto:

from numpy import * 
c=arcsin((2-sqrt(2*pi**2-4))/(2*pi))
derivada_c = sin(c)+cos(c)
k=2/pi
print('El valor de c es: {c}'.format(c=c))
## El valor de c es: -0.3184557133984764

Ejemplo

print('El valor de la derivada de f en c es:{x}'.format(x=derivada_c))
## El valor de la derivada de f en c es:0.6366197723675814
print('El valor de 2/pi es:{k}'.format(k=k))
## El valor de 2/pi es:0.6366197723675814

Regla de L’Hôpital

Una de las aplicaciones mĂ¡s importantes de las derivadas es su aplicaciĂ³n al cĂ¡lculo de lĂ­mites de funciones.

La regla de L’Hôpital permite resolver indeterminaciones usando derivadas:

Regla de L’Hôpital

Teorema: regla de L’Hôpital.

Sean \(f,g:(a,b)\longrightarrow\mathbb{R}\) dos funciones derivables en un punto \(c\in (a,b)\). Supongamos:

  • \(f(c)=g(c)=0\),
  • \(g(x)\neq 0\) y \(g'(x)\neq 0\) para todo punto \(x\) del entorno de \(c\) diferente de \(c\),

Si se verifican las condiciones anteriores y el límite siguiente existe \(\displaystyle\lim_{x\to c}\frac{f'(x)}{g'(x)}\) y vale \(L\), entonces también existe el límite \(\displaystyle\lim_{x\to c}\frac{f(x)}{g(x)}\) y vale \(L\).

Regla de L’Hôpital

DemostraciĂ³n

Sea \(\delta >0\) tal que \((c-\delta,c+\delta)\subseteq (a,b)\) y donde se cumplen las condiciones del teorema.

Sea \(x\in (c-\delta,c+\delta)\) un punto del entorno. En el intervalo \([c,x]\) se cumplen las hipĂ³tesis del Teorema del valor medio de Cauchy, ya que \(f\) y \(g\) son derivables en \((c,x)\) al serlo en todo el entorno \((c-\delta,c+\delta)\), y continuas en \([c,x]\), ya que como son derivables en todo el entorno, serĂ¡n continuas y como \([c,x]\subset (c-\delta,c+\delta)\), tambiĂ©n serĂ¡n continuas en el intervalo \([c,x]\).

Usando por tanto el Teorema del valor medio de Cauchy, podemos afirmar que existe un punto \(d\in (c,x)\) tal que: \[ f'(d)\cdot (g(d)-g(c))=g'(d)\cdot (f(d)-f(c)). \] Recordemos que \(f(c)=g(c)=0\), \(f'(d)\neq 0\) y \(g'(d)\neq 0\) ya que suponĂ­amos que las derivadas \(f'\) y \(g'\) no se anulaban en el entorno de \(c\). Usando las condiciones anteriores, podemos simplificar la expresiĂ³n anterior de la siguiente manera: \[ \frac{f'(d)}{g'(d)}=\frac{f(d)}{g(d)}. \]

Regla de L’Hôpital

Si hacemos tender \(x\to c\), como \(d\in (c,x)\), tendremos que \(d\to c\). Por tanto, \[ \lim_{x\to c}\frac{f(x)}{g(x)}=\lim_{x\to c}\frac{f'(x)}{g'(x)}=L, \] por hipĂ³tesis.

En resumen, si existe \(\displaystyle \lim_{x\to c}\frac{f'(x)}{g'(x)}\), también existe el límite \(\displaystyle \lim_{x\to c}\frac{f(x)}{g(x)}\) y los dos coinciden.

ObservaciĂ³n.

El teorema sigue siendo cierto en el caso en que \(a\), \(b\), \(c\) o \(L\) son \(\pm \infty\). Es decir, si existe el límite \(\displaystyle \lim_{x\to \pm\infty}\frac{f'(x)}{g'(x)}\), también existe el límite \(\displaystyle \lim_{x\to \pm\infty}\frac{f(x)}{g(x)}\) y los dos coinciden y pueden tener como resultado \(\pm\infty\).

Ejemplo

Ejemplo

Calculemos el valor del lĂ­mite siguiente \(\displaystyle\lim_{x\to 0}\frac{\sin x-x}{x-x\cos x}\).

Observemos que si sustituimos por el valor \(0\), obtenemos la indeterminaciĂ³n \(\frac{0}{0}\).

Usando la regla de L’HĂ´pital, calculemos el lĂ­mite anterior: \[ \lim_{x\to 0}\frac{\sin x-x}{x-x\cos x} = \lim_{x\to 0}\frac{\cos x-1}{1-\cos x+x\sin x}=\frac{0}{0}. \] Nos vuelve a dar la indeterminaciĂ³n \(\frac{0}{0}\). Aplicando la regla de L’HĂ´pital por segunda vez, obtenemos: \[ \lim_{x\to 0}\frac{\cos x-1}{1-\cos x+x\sin x} =\lim_{x\to 0}\frac{-\sin x}{\sin x+\sin x+x\cos x}=\lim_{x\to 0}\frac{-\sin x}{2\sin x+x\cos x}=\frac{0}{0}. \]

Ejemplo

Nos vuelve a dar la indeterminaciĂ³n \(\frac{0}{0}\). Aplicando la regla de L’HĂ´pital por tercera vez, obtenemos: \[ \lim_{x\to 0}\frac{-\sin x}{2\sin x+x\cos x} =\lim_{x\to 0}\frac{-\cos x}{2\cos x+\cos x-x\sin x}=\frac{-1}{3}. \] El lĂ­mite tiene el valor \(-\frac{1}{3}\).

El lĂ­mite anterior en Wolfram Alpha se muestra en el enlace siguiente:

Regla de L’Hôpital

¡Cuidado!

La Regla de L’HĂ´pital sĂ³lo se puede aplicar en un sentido. Es decir, si existe el lĂ­mite \(\displaystyle \lim_{x\to c}\frac{f'(x)}{g'(x)}\), bajo las condiciones del teorema anterior, existe el \(\displaystyle \lim_{x\to c}\frac{f(x)}{g(x)}\) y son iguales; ahora bien, si no existe el lĂ­mite \(\displaystyle \lim_{x\to c}\frac{f'(x)}{g'(x)}\), no podemos decir nada acerca del lĂ­mite \(\displaystyle \lim_{x\to c}\frac{f(x)}{g(x)}\). VĂ©ase el ejemplo siguiente.

Ejemplo

Ejemplo

Consideremos el lĂ­mite siguiente: \(\displaystyle\lim_{x\to 0}\frac{x^2\sin\left(\frac{1}{x}\right)}{\sin x}.\)

Si sustituimos \(x\) por \(0\) en el lĂ­mite anterior obtenemos el valor \(\frac{0}{0}\), pensad que \(\displaystyle\lim_{x\to 0}x^2\sin\left(\frac{1}{x}\right)=0\) vale \(0\) ya que es el lĂ­mite de una funciĂ³ que tiende a 0 (\(x^2\)) por una funciĂ³n acotada \(\left(\sin\left(\frac{1}{x}\right)\right)\).

Apliquemos pues la regla de l’HĂ´pital: \[ \begin{array}{rl} \displaystyle\lim_{x\to 0}\frac{\left(x^2\sin\left(\frac{1}{x}\right)\right)'}{(\sin x)'} & \displaystyle =\lim_{x\to 0}\frac{2x\sin\left(\frac{1}{x}\right)+x^2\cdot \left(-\frac{1}{x^2}\cos\left(\frac{1}{x}\right)\right)}{\cos x}= \lim_{x\to 0}\frac{2x\sin\left(\frac{1}{x}\right)-\cos\left(\frac{1}{x}\right)}{\cos x} \\ & \displaystyle =\lim_{x\to 0} \frac{2x\sin\left(\frac{1}{x}\right)}{\cos x}-\lim_{x\to 0}\frac{\cos\left(\frac{1}{x}\right)}{\cos x} = 0-\lim_{x\to 0}\frac{\cos\left(\frac{1}{x}\right)}{\cos x} = -\lim_{x\to 0}\frac{\cos\left(\frac{1}{x}\right)}{\cos x}. \end{array} \] El lĂ­mite \(\displaystyle \lim_{x\to 0} \frac{2x\sin\left(\frac{1}{x}\right)}{\cos x}\) vale \(0\) ya que el denominador tiende a \(1\) cuando \(x\to 0\) y el numerador es el lĂ­mite de una funciĂ³n que tiende a \(0\) (\(2x\)) por una funciĂ³n acotada \(\left(\sin\left(\frac{1}{x}\right)\right)\).

Ejemplo

El lĂ­mite del cociente de derivadas no existe ya que si consideramos la sucesiĂ³n \(x_n=\frac{1}{2\pi n}\longrightarrow 0\), si \(n\to\infty\), el lĂ­mite de la sucesiĂ³n \(\frac{\cos\left(\frac{1}{x_n}\right)}{\cos x_n}\) vale: \[ -\lim_{n\to\infty}\frac{\cos (2\pi n)}{\cos\left(\frac{1}{2\pi n}\right)}=-1. \] En cambio, si consideramos la sucesiĂ³n \(y_n =\frac{1}{\frac{\pi}{2}+2\pi n}\longrightarrow 0\), si \(n\to\infty\), el lĂ­mite de la sucesiĂ³n \(\frac{\cos\left(\frac{1}{y_n}\right)}{\cos y_n}\) vale: \[ -\lim_{n\to\infty}\frac{\cos \left(\frac{\pi}{2}+2\pi n\right)}{\cos\left(\frac{1}{\frac{\pi}{2}+2\pi n}\right)}=0. \]

Ejemplo

A continuaciĂ³n estarĂ­amos tentados a decir que nuestro lĂ­mite inicial \(\displaystyle\lim_{x\to 0}\frac{x^2\sin\left(\frac{1}{x}\right)}{\sin x}\) no existe pero esto es falso ya que: \[ \lim_{x\to 0}\frac{x^2\sin\left(\frac{1}{x}\right)}{\sin x}= \lim_{x\to 0}\frac{x}{\sin x}\cdot \lim_{x\to 0} x\sin\left(\frac{1}{x}\right)=1\cdot 0=0! \] El primer lĂ­mite \(\displaystyle \lim_{x\to 0}\frac{x}{\sin x}\) vale \(1\) ya que vimos en su momento que \(\displaystyle \lim_{x\to 0}\frac{\sin x}{x}=1\), por tanto, si hacemos el lĂ­mite de su recĂ­proco tambiĂ©n serĂ¡ \(1\): \(\displaystyle \lim_{x\to 0}\frac{x}{\sin x}=1\).

El segundo lĂ­mite \(\displaystyle \lim_{x\to 0} x\sin\left(\frac{1}{x}\right)\) vale \(0\) ya que es el lĂ­mite de una funciĂ³n que tiende a \(0\) (\(x\)) por una funciĂ³n acotada \(\left(\sin\left(\frac{1}{x}\right)\right)\).

Ejemplo

En resumen, que no exista el límite una vez aplicada la regla de l’Hôpital no significa que no exista el límite inicial que nos hemos planteado.

El lĂ­mite anterior en Wolfram Alpha se muestra en el enlace siguiente:

FĂ³rmula de Taylor

IntroducciĂ³n

La idea fundamental de la fĂ³rmula de Taylor es aproximar localmente una funciĂ³n en un entorno de un valor determinado por las funciones mĂ¡s manejables que se conocen, los polinomios.

Dicho de manera mĂ¡s explĂ­cita, consideremos una funciĂ³n \(f:(a,b)\longrightarrow\mathbb{R}\) que se puede derivar hasta un cierto orden, pongamos \(n+1\), para un cierto valor \(n\) natural, y sea \(x_0\in (a,b)\) un punto del interior del dominio de \(f\). Queremos hallar un polinomio \(P_n(x)\) tal que se verifique que \(f\) y \(P_n\) sean “iguales” en \(x_0\) hasta orden \(n\): \[ \lim_{x\to x_0}\frac{f(x)-P_n(x)}{(x-x_0)^m} =0,\mbox{ para }m=0,1,\ldots,n. \]

IntroducciĂ³n

La condiciĂ³n anterior para \(m=0\) es la siguiente: \[ \lim_{x\to x_0}f(x)-P_n(x) =0,\ \Rightarrow P_n(x_0)=f(x_0), \] es decir, la funciĂ³n y el polinomio a hallar deben coincidir en el valor \(x_0\).

IntroducciĂ³n

La condiciĂ³n anterior para \(m=1\) es la siguiente: \[ \lim_{x\to x_0}\frac{f(x)-P_n(x)}{(x-x_0)} =0,\ \Rightarrow P_n'(x_0)=f'(x_0), \] es decir, la derivada de la funciĂ³n y el polinomio a hallar deben coincidir en el valor \(x_0\) ya que si aplicamos la regla de L’HĂ´pital (el lĂ­mite es indeterminado de la forma \(\frac{0}{0}\) ya que recordemos que \(P_n(x_0)=f(x_0)\)): \[ \lim_{x\to x_0}\frac{f(x)-P_n(x)}{(x-x_0)} = \lim_{x\to x_0}\frac{f'(x)-P_n'(x)}{1}=0. \]

IntroducciĂ³n

En general, la condiciĂ³n para \(m\) entre \(0\) y \(n\) es la siguiente: \[ \lim_{x\to x_0}\frac{f(x)-P_n(x)}{(x-x_0)^m} =0,\ \Rightarrow P_n^{(m)}(x_0)=f^{(m)}(x_0), \] es decir, la derivada \(m\)-Ă©sima de la funciĂ³n y el polinomio a hallar deben coincidir en el valor \(x_0\) ya que si aplicamos la regla de L’HĂ´pital \(m\) veces (el lĂ­mite es indeterminado de la forma \(\frac{0}{0}\) ya que recordemos que \(P_n^{(i)}(x_0)=f^{(i)}(x_0)\) para los \(i\) anteriores desde \(0\) hasta \(m-1\)): \[ \lim_{x\to x_0}\frac{f(x)-P_n(x)}{(x-x_0)^m} = \lim_{x\to x_0}\frac{f'(x)-P_n'(x)}{m (x-x_0)^{m-1}}=\cdots = \lim_{x\to x_0}\frac{f^{(m)}(x)-P_n^{(m)}(x)}{m!}=0. \]

IntroducciĂ³n

Importante:

Las condiciones que debe verificar el polinomio \(P_n(x)\) para aproximar la funciĂ³n \(f(x)\) hasta orden \(n\) en un entorno del punto \(x_0\) son las siguientes: \[ P_n^{(m)}(x_0)=f^{(m)}(x_0),\mbox{ para }m=0,\ldots,n. \] En este caso decimos que el polinomio \(P_n(x)\) tiene en el punto \(x_0\) orden de contacto con \(f\) superior a \(n\).

IntroducciĂ³n

Ejemplo ilustrativo

En el enlace siguiente se muestra la funciĂ³n \(f(x)=\sin x\) (en rojo) y los polinomios de grado \(1\), \(P_1(x)\) (la recta en azul discontinua), de grado \(3\), \(P_3(x)\) (la curva en azul discontinua) para \(x_0=0\). El polinomio \(P_2(x)\) coincide con el polinomio de grado \(1\) en este caso ya que el coeficiente de \(x^2\) vale \(0\) como veremos mĂ¡s adelante.

En la casilla expansion point podéis cambiar el valor \(x_0\). Intentad escribir pi/2 y pi y observad qué ocurre.

Si clicĂ¡is en la casilla More terms en la parte de arriba del grĂ¡fico verĂ©is los polinomios de grado \(5\), \(P_5(x)\) y de grado \(7\), \(P_7(x)\). Observad cĂ³mo cada vez los polinomios se aproximan mĂ¡s a la funciĂ³n \(f(x)\).

ObservaciĂ³n.

La elecciĂ³n del punto \(x_0\) no es arbitraria. Hemos de elegir un valor \(x_0\) del que conozcamos el valor \(f(x_0)\) y las derivadas de cualquier orden en \(x_0\), \(f^{(m)}(x_0)\), \(m=1,2,\ldots\)

IntroducciĂ³n

Ejemplo (continuaciĂ³n)

En el ejemplo anterior donde recordemos que \(f(x)=\sin x\), debemos elegir un punto \(x_0\) en el cual conozcamos los valores de \(\sin x_0\) y \(\cos x_0\) ya que si conocemos dichos valores, conoceremos \(f(x_0)\) y las derivadas de cualquier orden: \[ f(x_0)=\sin x_0,\ f'(x_0)=\cos x_0,\ f''(x_0)=-\sin x_0,\ f'''(x_0)=-\cos x_0,\ f^{iv}(x_0)=\sin x_0,\ldots \] Algunos valores \(x_0\) elegibles en este caso son los siguientes: \(x_0=0,\frac{\pi}{6}, \frac{\pi}{4},\frac{\pi}{3},\frac{\pi}{2},\pi\) ya que conocemos el valor de \(\sin (x_0)\) y \(\cos(x_0)\) tal como se observa en la tabla siguiente:

\(x_0\) \(0\) \(\frac{\pi}{6}\) \(\frac{\pi}{4}\) \(\frac{\pi}{3}\) \(\frac{\pi}{2}\) \(\pi\)
\(\sin(x_0)\) \(0\) \(\frac{1}{2}\) \(\frac{\sqrt{2}}{2}\) \(\frac{\sqrt{3}}{2}\) \(1\) \(0\)
\(\cos(x_0)\) \(1\) \(\frac{\sqrt{3}}{2}\) \(\frac{\sqrt{2}}{2}\) \(\frac{1}{2}\) \(0\) \(-1\)

CĂ¡lculo del polinomio de Taylor

El resultado siguiente nos da una expresiĂ³n del polinomio de Taylor:

Teorema. ExpresiĂ³n del polinomio de Taylor.

Sea \(n\) un valor natural. Sea \(f:(a,b)\longrightarrow\mathbb{R}\) una funciĂ³n real de variable real. Sea \(x_0\in (a,b)\) un punto interior del dominio de \(f\). Supongamos que \(f\) es derivable \(n+1\) veces en \(x_0\). Entonces el polinomio de Taylor de grado \(n\) con orden de contacto con \(f\) superior a \(n\) en \(x_0\) es el siguiente: \[ \begin{array}{rl} P_n(x) = & f(x_0)+f'(x_0)\cdot (x-x_0)+\frac{f''(x_0)}{2!}\cdot (x-x_0)^2+\cdots +\\ & +\frac{f^{(n)}(x_0)}{n!}\cdot (x-x_0)^n. \end{array} \]

CĂ¡lculo del polinomio de Taylor

Contenido bastante técnico.

DemostraciĂ³n

Vamos a demostrar la fĂ³rmula anterior por inducciĂ³n sobre \(n\).

Para \(n=0\), \(P_0(x)=f(x_0)\), que por definiciĂ³n es el polinomio de Taylor de grado \(0\) o constante.

Suponemos cierto para \(n\), es decir suponemos que: \[ P_n(x) = f(x_0)+f'(x_0)\cdot (x-x_0)+\frac{f''(x_0)}{2!}\cdot (x-x_0)^2+\cdots +\frac{f^{(n)}(x_0)}{n!}\cdot (x-x_0)^n. \] Hemos de demostrar que: \[ \begin{array}{rl} P_{n+1}(x) = & f(x_0)+f'(x_0)\cdot (x-x_0)+\frac{f''(x_0)}{2!}\cdot (x-x_0)^2+\cdots + \frac{f^{(n)}(x_0)}{n!}\cdot (x-x_0)^n+\\ & +\frac{f^{(n+1)}(x_0)}{(n+1)!}\cdot (x-x_0)^{n+1} = P_n(x)+\frac{f^{(n+1)}(x_0)}{(n+1)!}\cdot (x-x_0)^{n+1}. \end{array} \]

CĂ¡lculo del polinomio de Taylor

Por hipĂ³tesis de inducciĂ³n, sabemos que: \(P_n^{(i)}(x_0)=f^{(i)}(x_0)\), para \(i=0,\ldots,n\) ya que recordemos que \(P_n(x)\) es el polinomio de Taylor de grado \(n\).

Para verificar que \(P_{n+1}(x)\) correspondiente a la expresiĂ³n anterior es el polinomio de Taylor de grado \(n+1\), hay que verificar las igualdades siguientes: \(P_{n+1}^{(i)}(x_0)=f^{(i)}(x_0)\), para \(i=0,\ldots,n+1\).

Si \(i\) estĂ¡ entre \(0\) y \(n\), tenemos que: \[ P_{n+1}^{(i)}(x)=P_{n}^{(i)}(x)+(n+1)\cdots (n+2-i)\cdot\frac{f^{(n+1)}(x_0)}{(n+1)!}\cdot (x-x_0)^{n+1-i}. \] Si evaluamos la expresiĂ³n anterior en \(x=x_0\), obtenemos: \[ P_{n+1}^{(i)}(x_0)=P_{n}^{(i)}(x_0)+(n+1)\cdots (n+2-i)\cdot\frac{f^{(n+1)}(x_0)}{(n+1)!}\cdot (x_0-x_0)^{n+1-i} = P_n^{(i)}(x_0)=f^{(i)}(x_0), \] ya que \(n+1-i>0\) por lo que el segundo sumando serĂ¡ nulo y la Ăºltima igualdad es cierta por hipĂ³tesis de inducciĂ³n.

CĂ¡lculo del polinomio de Taylor

SĂ³lo falta demostrar el caso \(i=n+1\). Si calculamos la derivada \(n+1\)-Ă©sima del polinomio \(P_{n+1}(x)\) obtenemos: \[ P_{n+1}^{(n+1)}(x)=P_n^{(n+1)}(x)+f^{(n+1)}(x_0) = f^{(n+1)}(x_0), \] ya que \(P_n^{(n+1)}(x)=0\) al ser \(P_n(x)\) un polinomio de grado \(n\) y por tanto la derivada \(n+1\)-Ă©sima del mismo serĂ¡ 0.

Concluimos por tanto que la derivada \(n+1\)-Ă©sima del polinomio \(P_{n+1}(x)\) serĂ¡ la constante \(f^{(n+1)}(x_0)\) y, en particular, se cumplirĂ¡ que \(P_{n+1}^{(n+1)}(x_0)=f^{(n+1)}(x_0)\), tal como querĂ­amos demostrar.

Ejemplo

Ejemplo

Consideremos la funciĂ³n \(f(x)=\sin (x)\) y el punto \(x_0=0\). Vamos a hallar el polinomio de Taylor de grado \(n\) de \(f(x)\) en \(x_0=0\).

Lo primero que hemos de calcular a la vista de la expresiĂ³n vista en el teorema anterior que nos da la expresiĂ³n del polinomio de Taylor es el valor de la funciĂ³n en \(x_0\), \(f(x_0)\) y el valor de las derivadas de \(f\) en \(x_0\), \(f^{(m)}(x_0)\), para \(m=1,2,\ldots\)

Los valores de \(f^{(m)}(0)\) valen lo siguiente: \[ f(0)=\sin 0=0,\ f'(0)=\cos 0=1,\ f''(0)=-\sin 0=0,\ f'''(0)=-\cos 0 =-1,\ f^{(iv)}(0)=\sin 0 =0, \ldots \] A partir de los cĂ¡lculos anteriores podemos deducir que \(f^{(n)}(0)=0\), si \(n\) es par y \(f^{(n)}(0)=\pm 1\), si \(n\) es impar y valdrĂ¡ \(1\) si \(n=1,5,9,\ldots\) y \(-1\) si \(n=3,7,11,\ldots\)

Intentemos escribir el resultado anterior de forma mĂ¡s “compacta”. Decir que \(n\) es par es equivalente a decir que existe un valor \(k\) natural tal que \(n=2k\) y decir que \(n\) es impar es equivalente a decir que existe un valor \(k\) natural tal que \(n=2k+1\). Por tanto, las condiciones anteriores se pueden escribir como: \(f^{(2k)}(0)=0\), \(f^{(2k+1)}(0)=\pm 1\).

Ejemplo

Observemos ademĂ¡s que los valores de \(n\) para los que la derivada \(n\)-Ă©sima valĂ­a \(1\), (\(n=1,5,9,\ldots\)) corresponde a valores de \(k\) par ya que \(1=2\cdot 0+1,\ 5=2\cdot 2+1,\ 9=2\cdot 4+1,\ldots\) y los valores de \(n\) para los que la derivada \(n\)-Ă©sima valĂ­a \(-1\) (\(n=3,7,11,\ldots\)) corresponde a valores de \(k\) impar ya que \(3=2\cdot 1+1,\ 7=2\cdot 3+1,\ 11=2\cdot 5+1,\ldots\)

Por tanto, la condiciĂ³n \(f^{(2k+1)}(0)=\pm 1\) puede escribirse como \(f^{(2k+1)}(0)=(-1)^k\) ya que la expresiĂ³n \((-1)^k\) da \(1\) para los \(k\) pares y \(-1\), para los \(k\) impares.

En resumen, tenemos lo siguiente: \(f^{(2k)}(0)=0\), \(f^{(2k+1)}(0)=(-1)^k\), para \(k=0,1,2,3,\ldots\)

Sea \(n\) un natural. Consideremos dos casos:

  • \(n\) par. En este caso, el polinomio de Taylor de grado \(n\) es el siguiente: \[ P_n(x)=f(0)+f'(0)\cdot x+\frac{f''(0)}{2!}\cdot x^2+\cdots + \frac{f^{n}(0)}{n!}\cdot x^n. \] Observemos que el Ăºltimo tĂ©rmino del polinomio anterior serĂ¡ \(0\) ya que hemos dicho que \(f^{(n)}(0)=0\) para \(n\) par. Entonces el polinomio \(P_n(x)\) se puede escribir como: \[ P_n(x)=f(0)+f'(0)\cdot x+\frac{f''(0)}{2!}\cdot x^2+\cdots + \frac{f^{n-1}(0)}{(n-1)!}\cdot x^{n-1}. \]

Ejemplo

Si eliminamos los tĂ©rminos correspondientes a derivadas pares al ser nulos, nos queda la expresiĂ³n siguiente: \[ P_n(x)=x-\frac{x^3}{3!}+\frac{x^5}{5!}+\cdots + \frac{(-1)^k x^{2k+1}}{(2k+1)!}=\sum_{i=0}^k \frac{(-1)^i x^{2i+1}}{(2i+1)!}, \] donde \(k\) es tal que \(n-1=2k+1\), o, lo que es lo mismo, \(k=\frac{n-2}{2}\).

Consideremos por ejemplo \(n=14\), en este caso \(k=\frac{14-2}{2}=6\). El polinomio de Taylor de \(f(x)=\sin x\) de grado \(14\) en \(x_0=0\) es el siguiente: \[ \begin{array}{rl} P_{14}(x) & =x-\frac{x^3}{3!}+\frac{x^5}{5!}-\frac{x^7}{7!}+\frac{x^9}{9!}-\frac{x^{11}}{11!}+\frac{x^{13}}{13!},\\ & = x-\frac{x^3}{6}+\frac{x^5}{120}-\frac{x^7}{5040}+\frac{x^9}{362880}-\frac{x^{11}}{39916800}+\frac{x^{13}}{6227020800}. \end{array} \]

Ejemplo

  • \(n\) impar. En este caso, el polinomio de Taylor de grado \(n\) es el siguiente: \[ P_n(x)=f(0)+f'(0)\cdot x+\frac{f''(0)}{2!}\cdot x^2+\cdots + \frac{f^{n}(0)}{n!}\cdot x^n. \] Si eliminamos los tĂ©rminos correspondientes a derivadas pares al ser nulos, nos queda la expresiĂ³n siguiente: \[ P_n(x)=x-\frac{x^3}{3!}+\frac{x^5}{5!}+\cdots + \frac{(-1)^k x^{2k+1}}{(2k+1)!}=\sum_{i=0}^k \frac{(-1)^i x^{2i+1}}{(2i+1)!}, \] donde \(k\) es tal que \(n=2k+1\), o, lo que es lo mismo, \(k=\frac{n-1}{2}\). Consideremos por ejemplo \(n=11\), en este caso \(k=\frac{11-1}{2}=5\). El polinomio de Taylor de \(f(x)=\sin x\) de grado \(11\) en \(x_0=0\) es el siguiente: \[ P_{11}(x) =x-\frac{x^3}{3!}+\frac{x^5}{5!}-\frac{x^7}{7!}+\frac{x^9}{9!}-\frac{x^{11}}{11!} = x-\frac{x^3}{6}+\frac{x^5}{120}-\frac{x^7}{5040}+\frac{x^9}{362880}-\frac{x^{11}}{39916800}. \] Si vĂ¡is al enlace siguiente y apretĂ¡is una vez la casilla More terms en la secciĂ³n Series expansion at x=0 os aparecerĂ¡ el polinomio de Taylor de grado 11 y si volvĂ©is a apretar, os aparecerĂ¡ el polinomio de Taylor de grado 19 que “incluye” el polinomio de Taylor de grado 14.

Desarrollos de MacLaurin

Ejercicio

Hallar el polinomio de Taylor de grado \(n\) para la misma funciĂ³n que el ejemplo anterior en el punto \(x_0=\frac{\pi}{2}\).

DefiniciĂ³n.

Dada una funciĂ³n \(f:(a,b)\longrightarrow \mathbb{R}\), \(n+1\) veces derivable tal que \(0\in (a,b)\) y sea \(P_n(x)\) el polinomio de Taylor de grado \(n\) en \(x_0=0\). Dicho polinomio se denomina polinomio o expansiĂ³n de MacLaurin de grado \(n\) de \(f\) .

En el ejemplo anterior, hemos hallado el polinomio de MacLaurin de grado \(n\) de la funciĂ³n \(f(x)=\sin x\).

Error en el polinomio de Taylor

Una vez conocido cĂ³mo hallar el polinomio de Taylor de una funciĂ³n \(f(x)\) en un punto \(x_0\) de su dominio, podemos usar dicho polinomio o dicha expansiĂ³n para aproximar el valor de dicha funciĂ³n \(f(x)\) para valores \(x\) cercanos a \(x_0\).

Ahora bien, si no tenemos manera de estimar o calcular alguna cota del error que estamos cometiendo, dicha aproximaciĂ³n no tiene ningĂºn sentido ya que serĂ­a como ir a ciegas, es decir, no sabemos hasta quĂ© punto el valor \(P_n(x)\) aproxima bien o no el valor de \(f(x)\).

El siguiente resultado nos da una expresiĂ³n que permite acotar el error cometido usando el polinomio de Taylor.

Error en el polinomio de Taylor

Teorema. Error en la fĂ³rmula de Taylor.

Sea \(f\) una funciĂ³n \(f:(a,b)\longrightarrow \mathbb{R}\), sea \(x_0\in (a,b)\) un punto interior del dominio de \(f\) y supongamos que \(f\) es \(n+1\) veces derivable en un entorno de \(x_0\). Sea \(P_n(x)\) el polinomio de Taylor de grado \(n\) en \(x_0\). Entonces si \(x\) es un punto del entorno anterior de \(x_0\), se verifica: \[ f(x)-P_n(x)=\frac{f^{n+1}(c)}{m\cdot n!}\cdot (x-c)^{n-m+1}\cdot (x-x_0)^m, \] donde \(c\) es un punto que estĂ¡ situado entre \(x_0\) y \(x\) (o entre \(x\) y \(x_0\), dependiendo de cuĂ¡l de los dos valores es el menor y el mayor) y se denota por \(x\in <x_0,x>\) y \(m\) es un natural entre \(1\) y \(n+1\). Dicha expresiĂ³n es conocida por resto de Cauchy.

Error en el polinomio de Taylor

ObservaciĂ³n.

A la expresiĂ³n \(f(x)-P_n(x)\) se le denota por \(R_n(x-x_0)\), \(R\) de resto.

Corolario.

En las condiciones del teorema anterior, considerando \(m=n+1\), tenemos la expresiĂ³n siguiente conocida como resto de Lagrange: \[ f(x)-P_n(x)=\frac{f^{n+1}(c)}{(n+1)!}\cdot (x-x_0)^{n+1}. \]

Esta es la expresiĂ³n mĂ¡s conocida de la expresiĂ³n del error del polinomio de Taylor de grado \(n\).

Error en el polinomio de Taylor

Contenido muy técnico.

DemostraciĂ³n

Sea \(x\in (a,b)\) un valor del interior del entorno de \(x_0\) donde \(f\) es \(n+1\) veces derivable. Fijado dicho valor de \(x\), se considera la funciĂ³n siguiente que depende de la variable \(t\): \[ F(t)=f(t)+\sum_{k=1}^n \frac{f^{(k)}(t)}{k!}\cdot (x-t)^k. \] Dicha funciĂ³n \(F(t)\) es continua y derivable en el intervalo \(<x_0,x>\) (recordemos que dicha expresiĂ³n vale \((x_0,x)\), si \(x>x_0\) y \((x,x_0)\) si \(x<x_0\)) ya que es suma de productos de continuas y derivables: pensad que \(f\) es derivable por hipĂ³tesis, \(f^{(k)}\) serĂ¡ derivable ya que \(k\leq n\) y por ser \(f\) derivable \(n+1\) veces por hipĂ³tesis y la funciĂ³n \((x-t)^k\) serĂ¡ derivable al ser un polinomio en \(t\).

Error en el polinomio de Taylor

Hallemos el valor de la derivada de \(F\), \(F'(t)\): \[ \begin{array}{rl} F'(t) = & \displaystyle f'(t)+\sum_{k=1}^n \frac{f^{(k+1)}(t)}{k!}\cdot (x-t)^k -\sum_{k=1}^n k\cdot \frac{f^{(k)}(t)}{k!}(x-t)^{k-1} \\ = &\displaystyle f'(t)+\sum_{k=1}^n \frac{f^{(k+1)}(t)}{k!}\cdot (x-t)^k -\sum_{k=1}^n \frac{f^{(k)}(t)}{(k-1)!}(x-t)^{k-1}\\ = & f'(t)+\frac{f''(t)}{1!}\cdot (x-t)+\frac{f'''(t)}{2!}\cdot (x-t)^2+\cdots+ \frac{f^{(n)}(t)}{(n-1)!}\cdot (x-t)^{n-1} + \frac{f^{(n+1)}(t)}{n!}\cdot (x-t)^n\\ & -\left(f'(t)+\frac{f''(t)}{1!}\cdot (x-t)+\frac{f'''(t)}{2!}\cdot (x-t)^2 +\cdots + \frac{f^{(n)}(t)}{(n-1)!}\cdot (x-t)^{n-1} \right) = \frac{f^{(n+1)}(t)}{n!}\cdot (x-t)^n. \end{array} \] Consideremos ahora una funciĂ³n \(G\) cualquiera continua en el intervalo \(<x_0,x>\) cerrado y diferenciable en el mismo intervalo anterior pero abierto tal que \(G'(t)\neq 0\) para todo \(t\in <x_0,x>\) y \(G(x_0)\neq G(x)\).

Si aplicamos el Teorema del valor medio de Cauchy al intervalo \(<x_0,x>\) a las funciones \(F\) y \(G\), tenemos que existe un valor \(c\in <x_0,x>\) tal que: \[ G'(c)\cdot (F(x)-F(x_0))=F'(c)\cdot (G(x)-G(x_0)). \]

Error en el polinomio de Taylor

El valor de \(F(x)-F(x_0)\) es precisamente el error que cometemos al aproximar \(f(x)\) por el polinomio de Taylor de grado \(n\), \(P_n(x)\) ya que: \[ F(x)-F(x_0)=f(t)-\left(f(x_0)+\sum_{k=1}^n \frac{f^{(k)}(x_0)}{k!}\cdot (x-x_0)^k\right)=f(x)-P_n(x). \] Usando la expresiĂ³n deducida del Teorema del valor medio de Cauchy, podemos escribir que: \[ F(x)-F(x_0)=f(x)-P_n(x)=R_n(x-x_0)=\frac{F'(c)}{G'(c)}\cdot (G(x)-G(x_0)). \] Diferentes expresiones del error se obtienen eligiendo \(G\) de una determinada forma:

  • Si \(G(t)=(x-t)^{n+1}\), tenemos que \(G'(t)=-(n+1)\cdot (x-t)^n\) y, por tanto: \[ \begin{array}{rl} R_n(x-x_0) & =\frac{F'(c)}{G'(c)}\cdot (G(x)-G(x_0))=\frac{\frac{f^{(n+1)}(c)}{n!}\cdot (x-c)^n}{-(n+1)\cdot (x-c)^n}\cdot \left(-(x-x_0)^{n+1}\right)\\ & =\frac{f^{(n+1)}(c)}{(n+1)!}\cdot (x-x_0)^{n+1}. \end{array} \]

Error en el polinomio de Taylor

  • Si \(G(t)=(x-t)^{m}\), con \(m\) natural entre \(1\) y \(n+1\), tenemos que \(G'(t)=-m\cdot (x-t)^{m-1}\) y, por tanto: \[ \begin{array}{rl} R_n(x-x_0) & =\frac{F'(c)}{G'(c)}\cdot (G(x)-G(x_0))=\frac{\frac{f^{(n+1)}(c)}{n!}\cdot (x-c)^n}{-m\cdot (x-c)^{m-1}}\cdot \left(-(x-x_0)^{m}\right)\\ & =\frac{f^{(n+1)}(c)}{m\cdot n!}\cdot (x-c)^{n-m+1}\cdot (x-x_0)^{m}, \end{array} \] tal como querĂ­amos demostrar.

Ejemplo

Ejemplo: cĂ¡lculo aproximado de \(\sin x\)

Vamos a intentar aproximar la funciĂ³n \(f(x)=\sin x\) para un \(x\) prĂ³ximo a \(0\).

Recordemos que el polinomio de Taylor (de hecho, el de MacLaurin) de la funciĂ³n \(f(x)\) de grado \(n\) es el siguiente: \[ P_n(x)=\sum_{i=0}^k \frac{(-1)^i x^{2i+1}}{(2i+1)!}, \] con \(k=\frac{n-2}{2}\), si \(n\) es par y \(k=\frac{n-1}{2}\), si \(n\) es impar.

El problema que nos planteamos es el siguiente: dado \(x\) y \(\epsilon\) un error absoluto mĂ¡ximo que estamos dispuestos a cometer, calcular el valor de \(P_n(x)\) tal que \(|f(x)-P_n(x)=|\sin x-P_n(x)|\leq \epsilon\).

El primer paso es calcular el valor de \(n\). Nos fijamos a partir de la expresiĂ³n de \(P_n(x)\) que si \(n\) es par el grado del polinomio de MacLaurin tiene grado \(n-1\) ya que la potencia mĂ¡s alta de \(x\), \(2k+1\) vale \(2k+1=n-1\).

Supondremos que \(n\) es par ya que tiene un término menos que si \(n\) es impar y esto es una ventaja a la hora de computar \(P_n(x)\).

Ejemplo

El error cometido \(R_n(x)\) usando el teorema anterior vale: (usaremos la fĂ³rmula de Lagrange) \[ f(x)-P_n(x)=R_n(x)=\frac{f^{n+1}(c)}{(n+1)!}\cdot x^{n+1}. \] Dicho error se puede acotar por: \[ |f(x)-P_n(x)|=|R_n(x)|=\left|\frac{f^{n+1}(c)}{(n+1)!}\cdot x^{n+1}\right|\leq \max_{c\in <0,x>}\left|f^{n+1}(c)\right|\cdot \frac{|x|^{n+1}}{(n+1)!}. \] Al ser \(n\) par, \(|f^{n+1}(c)|=|\cos c|\) ya que \(n+1\) es impar y recordemos que cualquier derivada impar era \(\pm \cos c\). Por tanto, podemos acotar \(\displaystyle \max_{c\in <0,x>}\left|f^{n+1}(c)\right|\) por \(1\): \(\displaystyle\max_{c\in <0,x>}\left|f^{n+1}(c)\right|\leq 1\) y la cota del error serĂ¡: \[ |f(x)-P_n(x)|=|R_n(x)|\leq \frac{|x|^{n+1}}{(n+1)!}. \] La \(n\) buscada debe verificar: \(\frac{|x|^{n+1}}{(n+1)!}\leq \epsilon\).

Ejemplo

Como para cualquier valor de \(x\) el lĂ­mite \(\displaystyle\lim_{n\to\infty} \frac{|x|^{n+1}}{(n+1)!}=0\), seguro que existe una \(n\) tal que \(\frac{|x|^{n+1}}{(n+1)!}\leq\epsilon\).

La funciĂ³n siguiente nos calcula la \(n\) dado \(x\) y el error en python asegurĂ¡ndose que \(n\) es par:

import math
def n(x,error):
 x=float(x)
 m=2
 while(abs(x)**(m+1)/math.factorial(m+1) >=error):
   m=m+1
 if(m % 2==1):
   m=m+1
 return(m)

Ejemplo

El valor de \(n\) para \(x=0.5\) con un error mĂ¡ximo permitido de \(0.0001\) serĂ¡:

n(0.5,0.0001)
## 6

El polinomio de Taylor serĂ­a en este caso: \[ P_6(x)=x-\frac{x^3}{3!}+\frac{x^5}{5!}. \] El valor de \(P_6(0.5)\) vale:

x=0.5
x-x**3/math.factorial(3)+x**5/math.factorial(5)
## 0.47942708333333334

Ejemplo

Sabemos que el valor anterior evalĂºa \(\sin 0.5\) con un error menor que \(0.0001\). ComprobĂ©moslo en python:

valor_pol_taylor = x-x**3/math.factorial(3)+x**5/math.factorial(5)
abs(sin(0.5)-valor_pol_taylor)
## 1.5447291303316568e-06

Ejemplo

Ejemplo: cĂ¡lculo de \(\mathrm{e}\)

Vamos a calcular \(\mathrm{e}\) con 6 cifras decimales exactas.

Para ello vamos a calcular el polinomio de Taylor de la funciĂ³n \(f(x)=\mathrm{e}^x\) para \(x_0=0\), \(P_n(x)\) y aproximaremos \(f(1)=\mathrm{e}\) por \(P_n(1)\) cometiendo un error menor que \(0.000001\).

Para calcular el polinomio de Taylor de \(f(x)=\mathrm{e}^x\), hemos de calcular \(f^{k}(x)\) para cualquier valor \(k\) natural. En este caso, observamos que \(f^{(k)}(x)=\mathrm{e}^x\) siempre vale lo mismo. Por tanto: \[ P_n(x)=\sum_{k=0}^n \frac{f^{(k)}(0)}{k!}\cdot x^k = \sum_{k=0}^n \frac{x^k}{k!}, \] ya que \(f^{(k)}(0)=\mathrm{e}^0=1.\)

Seguidamente vamos a calcular el valor que \(n\) que nos asegure que el error cometido para \(x=1\) usando la expresiĂ³n anterior \(P_n(1)\) en lugar de \(f(1)=\mathrm{e}\) es menor que \(e=0.000001\).

Ejemplo

Recordemos la expresiĂ³n de la fĂ³rmula del error: \[ |f(x)-P_n(x)|=|R_n(x)|=\left|\frac{f^{n+1}(c)}{(n+1)!}\cdot x^{n+1}\right|\leq \max_{c\in <0,x>}\left|f^{n+1}(c)\right|\cdot \frac{|x|^{n+1}}{(n+1)!}. \] La expresiĂ³n anterior para \(x=1\) vale: \[ \begin{array}{rl} |f(1)-P_n(1)| & =|R_n(1)|=\left|\frac{f^{n+1}(c)}{(n+1)!}\cdot 1^{n+1}\right|\leq \max_{c\in (0,1)}\left|f^{n+1}(c)\right|\cdot \frac{1}{(n+1)!}=\max_{c\in (0,1)}\mathrm{e}^c\cdot \frac{1}{(n+1)!}\\ & =\frac{\mathrm{e}}{(n+1)!}. \end{array} \] En la Ăºltima igualdad hemos usado que la funciĂ³n \(f(x)=\mathrm{e}^x\) es creciente y por tanto \(\displaystyle\max_{c\in (0,1)}\mathrm{e}^c=\mathrm{e}^1=\mathrm{e}\).

Vemos que la cota del error depende del valor de \(\mathrm{e}\) que es precisamente el valor que queremos calcular.

No sabemos el valor exacto de \(\mathrm{e}\) pero podemos usar que es menor que \(3\): \(\mathrm{e}<3\).

La cota anterior serĂ¡, pues: \[ |f(1)-P_n(1)|=|R_n(1)|=\frac{\mathrm{e}}{(n+1)!}<\frac{3}{(n+1)!}. \]

Ejemplo

La funciĂ³n siguiente nos calcula la \(n\) dado el error en python:

def ne(error):
 m=2
 while(3./math.factorial(m+1) >=error):
   m=m+1
 return(m)

El valor de \(n\) para un error de \(0.000001\) vale:

ne(0.000001)
## 9

Ejemplo

El valor de \(\mathrm{e}\) con 6 cifras decimales exactas serĂ¡: \[ \mathrm{e}\approx 1+1+\frac{1}{2!}+\frac{1}{3!}+\frac{1}{4!}+\frac{1}{5!}+\frac{1}{6!}+\frac{1}{7!}+\frac{1}{8!}+\frac{1}{9!}. \] Si calculamos su valor en python, obtenemos:

valor_e_aproximado =1
for i in range(1,10):
  valor_e_aproximado=valor_e_aproximado+1./math.factorial(i)
valor_e_aproximado
## 2.7182815255731922

Comprobamos que efectivamente tiene 6 cifras decimales exactas:

math.exp(1)
## 2.718281828459045

Ejemplo

Ejemplo: generalizaciĂ³n del binomio de Newton

Recordemos la fĂ³rmula del binomio de Newton: dados \(x,y\in\mathbb{R}\) y un natural \(N\), podemos desarrollar la potencia \((x+y)^N\) como: \[ (x+y)^N = \sum_{k=0}^N \binom{N}{k}\cdot x^k\cdot y^{N-k}. \] Sea ahora la funciĂ³n \(f(x)=(x+C)^N\), donde \(C\) es una constante cualquiera. Sea \(x_0\) un valor real. Queremos hallar el polinomio de Taylor de la funciĂ³n anterior alrededor de \(x=x_0\).

Usando la fĂ³rmula del binomio de Newton anterior, el polinomio de Taylor de \(f(x)\) de grado \(N\) alrededor del valor \(x=x_0\) es relativamente sencillo de obtener: \[ \begin{array}{rl} f(x)=(x+C)^N & \displaystyle =((x-x_0)+(C+x_0))=\sum_{k=0}^N \binom{N}{k}\cdot (x-x_0)^k\cdot (C+x_0)^{N-k}\\ & =\displaystyle \sum_{k=0}^N \binom{N}{k}\cdot (C+x_0)^{N-k}\cdot (x-x_0)^k. \end{array} \]

Ejemplo

Observad que el desarrollo anterior no tiene error ya que la expresiĂ³n de la izquierda es un polinomio de grado \(N\). Otra manera de verlo es usar la expresiĂ³n de la fĂ³rmula del error: \[ f(x)-P_N(x)=R_N(x-x_0)=\frac{f^{N+1}(c)}{(N+1)!}\cdot (x-x_0)^{N+1}. \] Ahora bien, como \(f(x)\) es un polinomio de grado \(N\), la derivada \(N+1\)-Ă©sima en cualquier valor serĂ¡ \(0\) (\(f^{N+1}(c)=0\)) y, por tanto, \(R_N(x-x_0)=0\).

Podemos hallar en particular una expresiĂ³n para \(f^{(k)}(x_0)\): \[ \begin{array}{rl} \frac{f^{(k)}(x_0)}{k!} & =\binom{N}{k}\cdot (C+x_0)^{N-k},\\ f^{(k)}(x_0) & =\binom{N}{k}\cdot k!\cdot (C+x_0)^{N-k} = N\cdot (N-1)\cdots (N-k+1)\cdot (C+x_0)^{N-k}. \end{array} \]

Ejemplo

Supongamos ahora que “generalizamos” la funciĂ³n \(f\) de la forma siguiente \(f(x)=(x+C)^\alpha\), donde \(\alpha\) no tiene por quĂ© ser entero sino cualquier valor real. Sea \(x_0\) un valor real que supondremos distinto de \(-C\) para no tener problemas en caso en que \(\alpha <0\) ya que en este caso \(\displaystyle\lim_{x\to x_0}f(x)=\lim_{x\to x_0}(x+C)^\alpha = \lim_{x\to x_0} 0^\alpha =\lim_{x\to x_0}\frac{1}{0^{-\alpha}}=\frac{1}{0}=\infty\).

¿CuĂ¡l serĂ­a el polinomio de Taylor de grado \(n\) para dicha funciĂ³n \(f(x)\) en \(x=x_0\)?

EstarĂ­amos tentados de generalizar la fĂ³rmula anterior de la forma siguiente: \[ P_n(x)=\sum_{k=0}^n \binom{\alpha}{k}\cdot (C+x_0)^{\alpha -k}\cdot (x-x_0)^k, \] pero ¿quĂ© vale \(\binom{\alpha}{k}\)?. Pensad que sabemos calcular \(\binom{N}{k}\) si \(N\) es natural pero ahora “nuestra” \(N\) es \(\alpha\) y es un valor real cualquiera.

Como \(\binom{N}{k}\) se puede escribir como \(\binom{N}{k}=\frac{N\cdot (N-1)\cdots (N-k+1)}{k!}\), podrĂ­amos generalizar \(\binom{\alpha}{k}\) como \(\binom{\alpha}{k}=\frac{\alpha\cdot (\alpha-1)\cdots (\alpha-k+1)}{k!}\) y la expresiĂ³n anterior ya tendrĂ­a sentido.

De acuerdo, todo lo escrito hasta ahora estĂ¡ muy bien, pero ¿es cierta la fĂ³rmula anterior?

Ejemplo

Veamos que sĂ­, que la fĂ³rmula anterior es cierta.

Primero calculemos las derivadas sucesivas de \(f(x)=(x+C)^\alpha\): \[ f'(x)=\alpha\cdot (x+C)^{\alpha-1},\ f''(x)=\alpha\cdot (\alpha-1)\cdot (x+C)^{\alpha-2},\ f'''(x)=\alpha\cdot (\alpha-1)\cdot (\alpha-2)\cdot (x+C)^{\alpha-3}, \] y, en general: \[ f^{(k)}(x)=\alpha\cdot (\alpha-1)\cdots (\alpha-k+1)\cdot (x+C)^{\alpha-k}. \] Si evaluamos en \(x=x_0\) obtenemos: \[ f^{(k)}(x_0)=\alpha\cdot (\alpha-1)\cdots (\alpha-k+1)\cdot (x_0+C)^{\alpha-k}=\binom{\alpha}{k}\cdot k!\cdot (x_0+C)^{\alpha-k}. \]

Ejemplo

El polinomio de Taylor de grado \(n\) de \(f(x)\) en \(x=x_0\) serĂ¡: \[ \begin{array}{rl} P_n(x) & \displaystyle =\sum_{k=0}^n \frac{f^{(k)}(x_0)}{k!}\cdot (x-x_0)^k = \sum_{k=0}^n\frac{\alpha\cdot (\alpha-1)\cdots (\alpha-k+1)}{k!}\cdot (x_0+C)^{\alpha-k}\cdot (x-x_0)^k \\ & \displaystyle = \sum_{k=0}^n \binom{\alpha}{k}\cdot (x_0+C)^{\alpha-k}\cdot (x-x_0)^k, \end{array} \] tal como querĂ­amos ver.

El error cometido serĂ¡: \[ R_n(x-x_0)=\frac{f^{(n+1)}(c)}{(n+1)!}\cdot (x-x_0)^{n+1}=\binom{\alpha}{n+1}\cdot (c+C)^{\alpha-n-1}\cdot (x-x_0)^{n+1} \] donde \(c\in <x_0,x>\).

Ejemplo

Como aplicaciĂ³n, calculemos el polinomio de Taylor de la funciĂ³n \(f(x)=\frac{1}{\sqrt{x+1}}\) para \(x_0=2\).

En este caso \(\alpha = -\frac{1}{2}\) y \(C=1\).

Los valores de \(\binom{-\frac{1}{2}}{k}\) son los siguientes: \[ \binom{-\frac{1}{2}}{k} = \frac{-\frac{1}{2}\cdot\left(-\frac{1}{2}-1\right)\cdots \left(-\frac{1}{2}-k+1\right)}{k!}=\frac{-\frac{1}{2}\cdot\left(-\frac{3}{2}\right)\cdots \left(-\frac{(2k-1)}{2}\right)}{k!}=\frac{(-1)^k\cdot (2k-1)!!}{2^k\cdot k!}. \] El polinomio de Taylor de grado \(n\) para \(f(x)\) en \(x_0=2\) serĂ¡: \[ P_n(x)=\sum_{k=0}^n \frac{(-1)^k\cdot (2k-1)!!}{2^k\cdot k!}\cdot 3^{-\frac{1}{2}-k}\cdot (x-2)^k=\sum_{k=0}^n \frac{(-1)^k\cdot (2k-1)!!}{2^k\cdot \sqrt{3^{2k+1}}\cdot k!}\cdot (x-2)^k. \] El error cometido serĂ¡: \[ R_n(x-2)=\binom{-\frac{1}{2}}{n+1}\cdot (1+c)^{-\frac{1}{2}-n-1}\cdot (x-2)^{n+1}=\frac{(-1)^{n+1}\cdot (2n+1)!!}{2^{n+1}\cdot \sqrt{(1+c)^{2n+3}}\cdot (n+1)!}\cdot (x-2)^{n+1}. \]

Ejemplo

Si suponemos que \(x>2\), usando que \(c\in (2,x)\), y, por tanto, \(\frac{1}{1+c}\leq \frac{1}{3}\), podemos acotar el error cometido por: \[ |R_n(x-2)|\leq\frac{(2n+1)!!}{2^{n+1}\cdot \sqrt{3^{2n+3}}\cdot (n+1)!}\cdot (x-2)^{n+1}. \] Consideremos \(x=2.25\). Calculemos el valor de \(n\) para calcular \(f(2.25)\) con un error menor que \(0.000001\):

def doublefactorial(n):
  if n in (0, 1):
    return 1
  else:
    return n * doublefactorial(n-2)

Ejemplo

def calculo_n(error):
  x=2.25
  m=2
  cota_error=(doublefactorial(2*m+1)/(2.**(m+1)*math.sqrt(3.)**(2*m+3)*
    math.factorial(m+1)))*(x-2)**(m+1) 
  while(cota_error >= error):
    m=m+1
    cota_error=(doublefactorial(2*m+1)/(2.**(m+1)*math.sqrt(3.)**(2*m+3)*
      math.factorial(m+1)))*(x-2)**(m+1) 
  return(m)

El valor del grado \(n\) del polinomio de Taylor serĂ¡:

calculo_n(0.000001)
## 4

Ejemplo

El polinomio de Taylor serĂ¡: \[ P_4(x)=\frac{1}{\sqrt{3}}-\frac{1}{6 \sqrt{3}}\cdot (x-2)+\frac{1}{24\sqrt{3}}\cdot (x-2)^2-\frac{5}{432\sqrt{3}}\cdot (x-2)^3+\frac{35}{10368\sqrt{3}}\cdot (x-2)^4. \] Calculemos el valor de \(P_4(2.25)\):

coeficientes=[1.,-1./6,1./24,-5./432,35./10368]
coeficientes=[(1./math.sqrt(3.))*c for c in coeficientes]
x=2.25
y=x-2
potencias=[1,y,y**2,y**3,y**4]
import numpy
numpy.dot(potencias,coeficientes)
## 0.5547007267350142

Ejemplo

Comprobemos que tiene efectivamente 6 cifras decimales exactas:

1/math.sqrt(2.25+1.)
## 0.5547001962252291

Propiedades del desarrollo de Taylor

La proposiciĂ³n siguiente es Ăºtil si queremos hallar el polinomio de Taylor de una funciĂ³n que puede escribirse como suma, producto o cociente de funciones donde es mĂ¡s fĂ¡cil conocer su polinomio de Taylor:

Sean \(f\), \(g\) funciones reales de variable real \(f,g:(a,b)\longrightarrow\mathbb{R}\) definidas en \((a,b)\) y sea \(x_0\in (a,b)\) un punto del interior de su dominio. Sean \(P_n(x)\) y \(Q_n(x)\) los polinomios de Taylor de grado \(n\) de las funciones \(f\) y \(g\), respectivamente en un entorno del punto \(x_0\).

Propiedades del desarrollo de Taylor

ProposiciĂ³n.

  • El polinomio de Taylor de toda funciĂ³n escrita como combinaciĂ³n lineal de \(f\) y \(g\), \(\lambda f+\mu g\), con \(\lambda,\mu\in \mathbb{R}\) constantes reales es \(\lambda P_n(x)+\mu Q_n(x)\).

  • La funciĂ³n producto \(f\cdot g\) admite un polinomio de Taylor en el mismo entorno del punto \(x_0\). Dicho polinomio de Taylor de la funciĂ³n \(f\cdot g\) se calcula realizando el producto de los dos polinomios, \(P_n(x)\cdot Q_n(x)\) y eliminando todos los tĂ©rminos de grado mayor o igual que \(n+1\) en \((x-x_0)\).

Propiedades del desarrollo de Taylor

ProposiciĂ³n.

  • Si \(g(x_0)\neq 0\), el cociente \(\frac{f}{g}\) admite un polinomio de Taylor en este entorno. Dicho polinomio de Taylor de la funciĂ³n \(\frac{f}{g}\) se calcula de realizar la operaciĂ³n \(\frac{P_n(x)}{Q_n(x)}\), desarrollando esta expresiĂ³n en potencias de \((x-x_0)\) y eliminando todos los tĂ©rminos de grado mayor o igual que \(n+1\).

Ejemplo

Ejemplo

Como ejemplo de aplicaciĂ³n, calculemos el polinomio de Taylor grado \(n\) de la funciĂ³n \(f(x)=\frac{1}{1-x^2}\) en un punto \(x_0\) cualquiera distinto de \(\pm 1\) ya que los valores \(\pm 1\) no pertenecen al dominio de \(f(x)\). Pensad que \(\displaystyle\lim_{x\to\pm 1}f(x)=\lim_{x\to\pm 1}\frac{1}{1-x^2}=\frac{1}{0}=\infty\).

Podemos descomponer la funciĂ³n \(f(x)\) de la forma siguiente: \[ \frac{1}{1-x^2}=\frac{1}{2}\left(\frac{1}{1-x}+\frac{1}{1+x}\right). \] Para calcular los polinomios de Taylor de las funciones \(f_1(x)=\frac{1}{1-x}\) y \(f_2(x)=\frac{1}{1+x}\) en un punto cualquiera \(x_0\), a los que llamaremos \(P_{1,n}(x)\) y \(P_{2,n}(x)\), respectivamente, podemos usar la tĂ©cnica vista en el ejemplo anterior donde generalizĂ¡bamos la fĂ³rmula de Newton.

Ejemplo

Escribimos \(f_1(x)\) de la forma siguiente: \(f_1(x)=-\frac{1}{x-1}\). Entonces el valor de \(\alpha\) vale \(\alpha=-1\) y \(C=-1\). El polinomio \(P_{1,n}(x)\) serĂ¡ el siguiente: \[ P_{1,n}(x)=-\sum_{k=0}^n\binom{-1}{k}\cdot (x_0-1)^{-1-k}\cdot (x-x_0)^k. \] Calculemos el valor de \(\binom{-1}{k}\): \[ \binom{-1}{k}=\frac{(-1)\cdot (-2)\cdots (-1-k+1)}{k!}=\frac{(-1)^k\cdot k!}{k!}=(-1)^k. \] El valor del polinomio \(P_{1,n}(x)\) serĂ¡ el siguiente: \[ P_{1,n}(x)=-\sum_{k=0}^n\frac{(-1)^k}{(x_0-1)^{k+1}}\cdot (x-x_0)^k. \]

Ejemplo

Usando el mismo razonamiento anterior, podemos obtener el polinomio \(P_{2,n}(x)\) donde \(\alpha =-1\) y \(C=1\): \[ P_{2,n}(x)=\sum_{k=0}^n\frac{(-1)^k}{(x_0+1)^{k+1}}\cdot (x-x_0)^k. \] Usando que \(f(x)=\frac{1}{2}\left(\frac{1}{1-x}+\frac{1}{1+x}\right)\), el polinomio de Taylor de la funciĂ³n \(f(x)\) en \(x_0\) de grado \(n\) serĂ¡: \[ P_n(x)=\frac{1}{2}\left(P_{1,n}(x)+P_{2,n}(x)\right)=\frac{1}{2}\sum_{k=0}^n (-1)^k\left(\frac{1}{(x_0+1)^{k+1}}-\frac{1}{(x_0-1)^{k+1}}\right)\cdot (x-x_0)^k. \]

En el enlace siguiente se muestra el desarrollo de Taylor de la funciĂ³n \(f(x)=\frac{1}{1-x^2}\) hasta orden \(5\), es decir, hasta el polinomio de grado \(5\), \(P_5(x)\). Si apretĂ¡is el botĂ³n More terms en la secciĂ³n Series expansion at x=x0, Mathematica os calcularĂ¡ mĂ¡s tĂ©rminos del desarrollo de Taylor o aumentarĂ¡ el grado del polinomio. Comparad los valores que da el Mathematica con la soluciĂ³n obtenida. En la secciĂ³n Series representation at x = x0 da una expresiĂ³n muy parecida a la obtenida.

Estudio local de una funciĂ³n

IntroducciĂ³n

Dada una funciĂ³n real de variable real \(f\), en esta secciĂ³n vamos a ver cĂ³mo podemos representarla grĂ¡ficamente analizando un conjunto de caracterĂ­sticas de la mismas, donde la mayorĂ­a de ellas estĂ¡n basadas en la funciĂ³n derivada.

Antes de realizar dicho estudio necesitamos:

  • analizar el crecimiento, decrecimento y extremos locales de la funciĂ³n \(f\) en el caso general, es decir, estudiando las derivadas de orden superior y,
  • analizar la concavidad y la convexidad de la funciĂ³n \(f\) que definiremos mĂ¡s adelante.

Crecimiento y extremos

El teorema siguiente nos dice cuĂ¡ndo una funciĂ³n es creciente o decreciente y nos da condiciones para los extremos a partir de condiciones que tienen que verificar derivadas de orden superior de la funciĂ³n:

Crecimiento y extremos

Teorema.

Sea \(f:(a,b)\longrightarrow\mathbb{R}\) una funciĂ³n real de variable real y sea \(x_0\in (a,b)\) un punto interior del dominio de \(f\). Supongamos que existe un natural \(n\) tal que \(f\) es derivable hasta orden \(2n+2\) en un entorno de \(x_0\). Entonces:

  • Si \(f'(x_0)=f''(x_0)=\cdots =f^{(2n)}(x_0)=0\) y \(f^{(2n+1)}(x_0)\neq 0\), resulta que:
    • si \(f^{(2n+1)}(x_0)>0\), \(f\) es creciente en \(x_0\),
    • si \(f^{(2n+1)}(x_0)<0\), \(f\) es decreciente en \(x_0\).
  • Si \(f'(x_0)=f''(x_0)=\cdots =f^{(2n-1)}(x_0)=0\) y \(f^{(2n)}(x_0)\neq 0\), resulta que:
    • si \(f^{(2n)}(x_0)>0\), entonces \(f\) tiene un mĂ­nimo en \(x=x_0\),
    • si \(f^{(2n)}(x_0)<0\), entonces \(f\) tiene un mĂ¡ximo en \(x=x_0\).

Crecimiento y extremos

ObservaciĂ³n.

Si aplicamos la primera parte del teorema anterior para \(n=0\), tenemos un resultado conocido: si \(f'(x_0)>0\), \(f\) es creciente en \(x_0\) y si \(f'(x_0)<0\), \(f\) es decreciente en \(x_0\). Por tanto, la primera parte del teorema generaliza dicho resultado.

Respecto a la segunda parte del teorema, sabĂ­amos que si \(f\) tiene un extremo relativo en \(x_0\), entonces \(f'(x_0)=0\) y tambiĂ©n sabĂ­amos que el recĂ­proco era falso. Esta segunda parte nos dice cuando dicho recĂ­proco es cierto. Si lo aplicamos en el caso sencillo de que \(n=1\), nos dice que si \(f''(x_0)>0\), entonces \((x_0,f(x_0))\) es un mĂ­nimo relativo de \(f\) y si \(f''(x_0)<0\), entonces \((x_0,f(x_0))\) es un mĂ¡ximo relativo de \(f\).

Crecimiento y extremos

Contenido bastante técnico.

DemostraciĂ³n

Consideremos la expresiĂ³n del polinomio de Taylor de grado \(2n+1\) en \(x_0\) junto con la expresiĂ³n del resto de Lagrange: \[ f(x)=\sum_{k=0}^{2n+1}\frac{f^{(k)}(x_0)}{k!}\cdot (x-x_0)^k + R_{2n+1}(x-x_0). \] Teniendo en cuenta que \(f'(x_0)=f''(x_0)=\cdots = f^{2n}(x_0)=0\), la expresiĂ³n anterior queda de la forma siguiente al ser \(0\) los sumandos para \(k=1,2,\ldots,2n\): \[ f(x)=f(x_0)+\frac{f^{(2n+1)}(x_0)}{(2n+1)!}\cdot (x-x_0)^{2n+1} + R_{2n+1}(x-x_0). \]

Crecimiento y extremos

Por tanto: \[ \frac{f(x)-f(x_0)}{(x-x_0)^{2n+1}}=\frac{f^{(2n+1)}(x_0)}{(2n+1)!} +\frac{R_{2n+1}(x-x_0)}{(x-x_0)^{2n+1}}. \]

Como \(R_{2n+1}(x-x_0)=\frac{f^{(2n+2)}(c)}{(2n+2)!}\cdot (x-x_0)^{2n+2}\), con \(c\in <x,x_0>\), se verificarĂ¡ que: \[ \lim_{x\to x_0}\frac{R_{2n+1}(x-x_0)}{(x-x_0)^{2n+1}}=\lim_{x\to x_0}\frac{f^{(2n+2)}(c)}{(2n+2)!}\cdot (x-x_0)=0. \] La condiciĂ³n anterior nos dice que el tĂ©rmino dominante en la expresiĂ³n de \(\frac{f(x)-f(x_0)}{(x-x_0)^{2n+1}}\) es el primero, \(\frac{f^{(2n+1)}(x_0)}{(2n+1)!}\), ya que el segundo, \(\frac{R_{2n+1}(x-x_0)}{(x-x_0)^{2n+1}}\), tiende a cero. Por tanto, para \(x\) suficientemente prĂ³ximo a \(x_0\), \[ \mathrm{signo}\left(\frac{f(x)-f(x_0)}{(x-x_0)^{2n+1}}\right)=\mathrm{signo}\left(\frac{f^{(2n+1)}(x_0)}{(2n+1)!}\right). \]

Crecimiento y extremos

Probemos a continuaciĂ³n las tesis de la primera parte del teorema:

  • supongamos que \(f^{(2n+1)}(x_0)>0\). En este caso, como \(\mathrm{signo}\left(\frac{f(x)-f(x_0)}{(x-x_0)^{2n+1}}\right)=\mathrm{signo}\left(\frac{f^{(2n+1)}(x_0)}{(2n+1)!}\right)\), tendremos que \(\frac{f(x)-f(x_0)}{(x-x_0)^{2n+1}} > 0\), y, como consecuencia \(\frac{f(x)-f(x_0)}{(x-x_0)} > 0\) (ya que \(2n+1\) es impar y el signo de \((x-x_0)^{2n+1}\) y \(x-x_0\) es el mismo), condiciĂ³n que equivale a decir que \(f\) es creciente en \(x_0\).

  • supongamos que \(f^{(2n+1)}(x_0)<0\). En este caso, como \(\mathrm{signo}\left(\frac{f(x)-f(x_0)}{(x-x_0)^{2n+1}}\right)=\mathrm{signo}\left(\frac{f^{(2n+1)}(x_0)}{(2n+1)!}\right)\), tendremos que \(\frac{f(x)-f(x_0)}{(x-x_0)^{2n+1}} < 0\), y, como consecuencia \(\frac{f(x)-f(x_0)}{(x-x_0)} < 0\) (por la misma razĂ³n que antes), condiciĂ³n que equivale a decir que \(f\) es decreciente en \(x_0\).

Crecimiento y extremos

Para la demostraciĂ³n de la segunda parte, consideremos la expresiĂ³n del polinomio de Taylor de grado \(2n\) en \(x_0\) junto con la expresiĂ³n del resto de Lagrange: \[ f(x)=\sum_{k=0}^{2n}\frac{f^{(k)}(x_0)}{k!}\cdot (x-x_0)^k + R_{2n}(x-x_0). \] Teniendo en cuenta que \(f'(x_0)=f''(x_0)=\cdots = f^{2n-1}(x_0)=0\), la expresiĂ³n anterior queda de la forma siguiente al ser \(0\) los sumandos para \(k=1,2,\ldots,2n-1\): \[ f(x)=f(x_0)+\frac{f^{(2n)}(x_0)}{(2n)!}\cdot (x-x_0)^{2n} + R_{2n}(x-x_0). \] Por tanto: \[ \frac{f(x)-f(x_0)}{(x-x_0)^{2n}}=\frac{f^{(2n)}(x_0)}{(2n)!} +\frac{R_{2n}(x-x_0)}{(x-x_0)^{2n}}. \]

Crecimiento y extremos

Como \(R_{2n}(x-x_0)=\frac{f^{(2n+1)}(c)}{(2n+1)!}\cdot (x-x_0)^{2n+1}\), con \(c\in <x,x_0>\), se verificarĂ¡ que: \[ \lim_{x\to x_0}\frac{R_{2n}(x-x_0)}{(x-x_0)^{2n}}=\lim_{x\to x_0}\frac{f^{(2n+1)}(c)}{(2n+1)!}\cdot (x-x_0)=0. \] La condiciĂ³n anterior nos dice que el tĂ©rmino dominante en la expresiĂ³n de \(\frac{f(x)-f(x_0)}{(x-x_0)^{2n}}\) es el primero, \(\frac{f^{(2n)}(x_0)}{(2n)!}\), ya que el segundo, \(\frac{R_{2n}(x-x_0)}{(x-x_0)^{2n}}\), tiende a cero. Por tanto, para \(x\) suficientemente prĂ³ximo a \(x_0\), \[ \mathrm{signo}\left(\frac{f(x)-f(x_0)}{(x-x_0)^{2n}}\right)=\mathrm{signo}\left(\frac{f^{(2n)}(x_0)}{(2n)!}\right). \]

Crecimiento y extremos

Probemos a continuaciĂ³n las tesis de la segunda parte del teorema:

  • si \(f^{(2n)}(x_0)>0\), como \(\mathrm{signo}\left(\frac{f(x)-f(x_0)}{(x-x_0)^{2n}}\right)=\mathrm{signo}\left(\frac{f^{(2n)}(x_0)}{(2n)!}\right)\), tendremos que \(\frac{f(x)-f(x_0)}{(x-x_0)^{2n}} > 0\), y, como consecuencia \(f(x)-f(x_0) > 0\) (ya que \(2n\) es par y el signo de \((x-x_0)^{2n}\) siempre es positivo), condiciĂ³n que equivale a decir que \(f\) tiene un mĂ­nimo en \(x_0\).

  • si \(f^{(2n)}(x_0)<0\), como \(\mathrm{signo}\left(\frac{f(x)-f(x_0)}{(x-x_0)^{2n}}\right)=\mathrm{signo}\left(\frac{f^{(2n)}(x_0)}{(2n)!}\right)\), tendremos que \(\frac{f(x)-f(x_0)}{(x-x_0)^{2n}} < 0\), y, como consecuencia \(f(x)-f(x_0) < 0\) (por la misma razĂ³n que antes), condiciĂ³n que equivale a decir que \(f\) tiene un mĂ¡ximo en \(x_0\).

Concavidad y convexidad

La concavidad/convexidad de una funciĂ³n es una propiedad muy Ăºtil a la hora de representar dicha funciĂ³n.

De forma intuitiva, una funciĂ³n convexa es aquĂ©lla que estĂ¡ por encima de las rectas tangentes en sus puntos de su grĂ¡fica y una funciĂ³n cĂ³ncava es aquĂ©lla que estĂ¡ por debajo.

Ver las dos grĂ¡ficas siguientes.

Concavidad y convexidad

Concavidad y convexidad

Concavidad y convexidad

DefiniciĂ³n.

Sea \(f:(a,b)\longrightarrow\mathbb{R}\) una funciĂ³n real de variable real. Diremos que \(f\) es convexa si, y sĂ³lo si, dados \(x_1,x_2\in (a,b)\) y \(t\in [0,1]\) entonces \[ f(t\cdot x_1+(1-t)\cdot x_2)\leq t\cdot f(x_1)+(1-t)\cdot f(x_2). \]

DefiniciĂ³n.

Sea \(f:(a,b)\longrightarrow\mathbb{R}\) una funciĂ³n real de variable real. Diremos que \(f\) es cĂ³ncava si, y sĂ³lo si, dados \(x_1,x_2\in (a,b)\) y \(t\in [0,1]\) entonces \[ f(t\cdot x_1+(1-t)\cdot x_2)\geq t\cdot f(x_1)+(1-t)\cdot f(x_2). \]

Diremos que \(f\) es estrictamente convexa o estrictamente cĂ³ncava si las desigualdades anteriores son estrictas.

Concavidad y convexidad

ObservaciĂ³n.

Las caracterĂ­sticas de concavidad y convexidad estĂ¡n definidas para todo un intervalos y, por tanto, son propiedades globales de la funciĂ³n \(f\).

GrĂ¡ficamente una funciĂ³n es convexa cuando dados dos valores cualesquiera \(x_1<x_2\) dentro del dominio de la funciĂ³n, el trozo de la grĂ¡fica de la funciĂ³n entre \(x_1\) y \(x_2\) estĂ¡ por debajo de la recta que une los puntos \((x_1,f(x_1))\) y \((x_2,f(x_2))\).

Se puede ver en la figura siguiente.

Concavidad y convexidad

Concavidad y convexidad

La condiciĂ³n anterior es equivalente a que para todo punto \(x\) del intervalo \([x_1,x_2]\), \(f(x)\) es menor o igual que la imagen de \(x\) por la recta que une los puntos \((x_1,f(x_1))\) y \((x_2,f(x_2)).\)

La ecuaciĂ³n de la recta que une los puntos \((x_1,f(x_1))\) e \((x_2,f(x_2))\) es la siguiente: \[ y=f(x_1)+\frac{f(x_2)-f(x_1)}{x_2-x_1}\cdot (x-x_1). \]

Concavidad y convexidad

Entonces decir que \(f(x)\) es menor o igual que la imagen de \(x\) por la recta que une los puntos \((x_1,f(x_1))\) y \((x_2,f(x_2))\) equivale a la expresiĂ³n siguiente: \[ f(x)\leq f(x_1)+\frac{f(x_2)-f(x_1)}{x_2-x_1}\cdot (x-x_1). \] Los puntos \(x\) que estĂ¡n en el intervalo \([x_1,x_2]\) se pueden escribir como \(t\cdot x_1+(1-t)\cdot x_2\) variando \(t\) en el intervalo \([0,1]\). Por ejemplo, para \(t=0\), obtenemos el extremo izquierdo \(x_1\), para \(t=1\), el extremo derecho \(x_2\) y para \(t=\frac{1}{2}\), el punto medio entre los dos valores \(\frac{x_1+x_2}{2}\).

Concavidad y convexidad

La condiciĂ³n de concavidad serĂ­a la siguiente: dados \(x_1< x_2\) y \(t\in [0,1]\) se verifica: \[ \begin{array}{rl} f(t\cdot x_1+(1-t)\cdot x_2) & \leq f(x_1)+\frac{f(x_2)-f(x_1)}{x_2-x_1}\cdot (t\cdot x_1+(1-t)\cdot x_2-x_1)\\ f(t\cdot x_1+(1-t)\cdot x_2) & \leq f(x_1)+\frac{f(x_2)-f(x_1)}{x_2-x_1}\cdot (1-t)\cdot (x_2-x_1)\\ f(t\cdot x_1+(1-t)\cdot x_2) & \leq f(x_1)+(f(x_2)-f(x_1))\cdot (1-t) \\ f(t\cdot x_1+(1-t)\cdot x_2) & \leq t\cdot f(x_1)+(1-t)\cdot f(x_2). \end{array} \] La Ăºltima desigualdad es la condiciĂ³n que tenemos en la definiciĂ³n de convexidad.

Concavidad y convexidad

La concavidad se razona de la misma manera, sĂ³lo se tiene que tener en cuenta que ahora dados dos valores cualesquiera \(x_1<x_2\) dentro del dominio de la funciĂ³n, el trozo de la grĂ¡fica de la funciĂ³n entre \(x_1\) y \(x_2\) estĂ¡ por encima de la recta que une los puntos \((x_1,f(x_1))\) y \((x_2,f(x_2))\).

Se puede ver en la figura siguiente.

Concavidad y convexidad

Concavidad y convexidad

La proposiciĂ³n siguiente nos dice que la recta tangente en un punto de la grĂ¡fica de una funciĂ³n convexa (cĂ³ncava) estĂ¡ por debajo (encima) de la funciĂ³n tal como enunciamos al introducir los conceptos de convexidad y concavidad:

ProposiciĂ³n.

Sea \(f:(a,b)\longrightarrow \mathbb{R}\) una funciĂ³n real de variable real. Suponemos que \(f\) es derivable en \((a,b)\) con derivada continua. Entonces,

  • \(f\) es convexa en \((a,b)\) si, y sĂ³lo si, se verifica que \(f(x)\geq f(\hat{x})+f'(\hat{x})\cdot (x-\hat{x})\), para todo valor \(x,\hat{x}\in (a,b)\) en dicho dominio.
  • si \(f\) es cĂ³ncava en \((a,b)\) si, y sĂ³lo si, se verifica que \(f(x)\leq f(\hat{x})+f'(\hat{x})\cdot (x-\hat{x})\), para todo valor \(x,\hat{x}\in (a,b)\) en dicho dominio.

Concavidad y convexidad

ObservaciĂ³n.

Las condiciones anteriores equivalen a decir que si \(f\) es convexa en un intervalo \((a,b)\) y \(x,\hat{x}\in (a,b)\), entonces la imagen de \(f\) en \(x\) es mayor que la imagen de \(x\) por la recta tangente de \(f\) en \(\hat{x}\). La recta tangente de \(f\) en \(\hat{x}\) tiene la ecuaciĂ³n siguiente: \(y=f(\hat{x})+f'(\hat{x})\cdot (x-\hat{x})\), por tanto, la imagen de \(x\) por la recta tangente serĂ¡: \(y=f(\hat{x})+f'(\hat{x})\cdot (x-\hat{x})\).

Entonces afirmar que \(f(x)\geq f(\hat{x})+f'(\hat{x})\cdot (x-\hat{x})\) equivale a afirmar que la funciĂ³n en \(x\) estĂ¡ por encima de la recta tangente en \(\hat{x}\) tal como se observaba en el grĂ¡fico anterior.

En el caso de que \(f\) sea cĂ³ncava, la observaciĂ³n anterior es la misma pero en lugar de decir que la funciĂ³n estĂ¡ por encima, hemos de decir que la funciĂ³n estĂ¡ por debajo.

Concavidad y convexidad

Contenido bastante técnico.

DemostraciĂ³n de la proposiciĂ³n

\(\Rightarrow\) Supongamos que \(f\) es convexa en \((a,b)\).

Sean \(x,\hat{x}\in (a,b)\) dos valores del dominio de \(f\). Por ser \(f\) convexa, se verifica la condiciĂ³n siguiente para todo valor \(t\in [0,1]\): \[ f(t\cdot x+(1-t)\cdot \hat{x})\leq t\cdot f(x)+(1-t)\cdot f(\hat{x}). \] Operando, obtenemos: \[ \frac{f(\hat{x}+t\cdot (x-\hat{x}))-f(\hat{x})}{t}\leq f(x)-f(\hat{x}). \]

Concavidad y convexidad

Como la desigualdad anterior es cierta para todo valor de \(t\in [0,1]\), si hacemos tender \(t\to 0\) la parte de la izquierda obtenemos: \[ \begin{array}{rl} \displaystyle \lim_{t\to 0}\frac{f(\hat{x}+t\cdot (x-\hat{x}))-f(\hat{x})}{t} & \leq f(x)-f(\hat{x})\\ \displaystyle f'(\hat{x})\cdot (x-\hat{x})& \leq f(x)-f(\hat{x}) \\ \displaystyle f(\hat{x})+f'(\hat{x})\cdot (x-\hat{x}) & \leq f(x), \end{array} \] tal como querĂ­amos ver.

El cĂ¡lculo del lĂ­mite \(\displaystyle\lim_{t\to 0}\frac{f(\hat{x}+t\cdot (x-\hat{x}))-f(\hat{x})}{t}\) se puede realizar usando la regla de L’HĂ´pital: \[ \begin{array}{rl} \displaystyle \lim_{t\to 0}\frac{f(\hat{x}+t\cdot (x-\hat{x}))-f(\hat{x})}{t} &\displaystyle =\lim_{t\to 0}\frac{\frac{d}{dt}(f(\hat{x}+t\cdot (x-\hat{x}))-f(\hat{x}))}{1} \\ & \displaystyle = \lim_{t\to 0}f'(\hat{x}+t\cdot (x-\hat{x}))\cdot (x-\hat{x}) =f'(\hat{x})\cdot (x-\hat{x}), \end{array} \] donde hemos aplicado la regla de la cadena al derivar respecto la variable \(t\).

El caso en que \(f\) sea cĂ³ncava se razona de manera similar.

Concavidad y convexidad

\(\Leftarrow\) Supongamos ahora que \(f(x)\geq f(\hat{x})+f'(\hat{x})\cdot (x-\hat{x})\), para todo valor \(x,\hat{x}\in (a,b)\) en dicho dominio.

Sea \(t\in [0,1]\). Entonces, como \(x_t = t\cdot x+(1-t)\cdot \hat{x}\in (a,b)\) podemos escribir que: \[ \begin{array}{rl} f(x) & \geq f(x_t)+f'(x_t)\cdot (x-x_t), \\ f(\hat{x}) & \geq f(x_t)+f'(x_t)\cdot (\hat{x}-x_t). \end{array} \] Multiplicando la primera desigualdad por \(t\) y la segunda por \(1-t\) y sumando, obtenemos: \[ t\cdot f(x)+(1-t)\cdot f(\hat{x})\geq f(x_t)+f'(x_t) (t\cdot (x-x_t)+(1-t)\cdot (\hat{x}-x_t)), \] pero \[ t\cdot (x-x_t)+(1-t)\cdot (\hat{x}-x_t) = t\cdot x+(1-t)\cdot \hat{x}-t\cdot x_t-(1-t)\cdot x_t=x_t-t\cdot x_t-x_t+t\cdot x_t =0. \] En conclusiĂ³n: \[ t\cdot f(x)+(1-t)\cdot f(\hat{x})\geq f(x_t)=f(t\cdot x+(1-t)\cdot \hat{x}), \] condiciĂ³n que equivale a decir que \(f\) es convexa en el intervalo \((a,b)\).

La demostraciĂ³n en el caso en que \(f\) sea cĂ³ncava se razona de manera similar.

Concavidad y convexidad

Un punto de inflexiĂ³n de una funciĂ³n es un punto donde a la izquierda del mismo la funciĂ³n tiene un tipo de convexidad (cĂ³ncava o convexa) y a la derecha, otro tipo de convexidad. Es decir, se pasa de cĂ³ncava a convexa o de convexa a cĂ³ncava:

DefiniciĂ³n de punto de inflexiĂ³n.

Sea \(f:(a,b)\longrightarrow\mathbb{R}\) una funciĂ³n real de variable real. Sea \(x_0\in (a,b)\) un punto del interior del dominio de \(f\). Diremos que \(x_0\) es un punto de inflexiĂ³n si existe un valor \(\delta >0\) tal que si \(x\in (x_0-\delta,x_0)\), \(f\) es concava (convexa) en \(x\) y si \(x\in (x_0,x_0+\delta)\), \(f\) es convexa (concava) en \(x\).

Se puede ver en la figura siguiente.

Concavidad y convexidad

Concavidad y convexidad

El siguiente resultado caracteriza la concavidad y la convexidad de una funciĂ³n a partir de sus derivadas:

Teorema.

Sea \(f:(a,b)\longrightarrow\mathbb{R}\) una funciĂ³n real de variable real. Sea \(x_0\in (a,b)\) un punto del interior del dominio de \(f\). Supongamos que \(f\) es derivable tres veces con \(f'''\) continua. Entonces:

  • si \(f''(x_0) \neq0\), entonces
    • si \(f''(x_0)>0\), tenemos que \(f\) es convexa en un entorno de \(x_0\) y
    • si \(f''(x_0)<0\), tenemos que \(f\) es cĂ³ncava en un entorno de \(x_0\).
  • si \(f''(x_0)=0\) y \(f'''(x_0)\neq 0\), entonces \(f\) tiene un punto de inflexiĂ³n en \(x=x_0\).

Concavidad y convexidad

ObservaciĂ³n.

Si aplicamos el teorema anterior, obtenemos que si \(f''(x_0)>0\), o, equivalentemente, que \(f'\) es una funciĂ³n creciente en \(x_0\), \(f\) es convexa en un entorno de \(x_0\) y si \(f''(x_0)<0\), o, equivalentemente, que \(f'\) es una funciĂ³n decreciente en \(x_0\), \(f\) es cĂ³ncava en un entorno de \(x_0\).

Concavidad y convexidad

En el grĂ¡fico que vimos al principio de la secciĂ³n se muestran en verde un conjunto de rectas tangentes a una funciĂ³n convexa en rojo. Fijaos que a medida que la variable \(x\) aumenta, la pendiente de las rectas tangentes, es decir, la derivada va aumentando. Es decir, las rectas tangentes van subiendo. Se empieza con rectas tangentes de pendiente negativa, se llega a la recta tangente de pendiente cero (el mĂ­nimo) y se acaba con rectas tangentes de pendiente positiva.

En el segundo grĂ¡fico que vimos al principio de la secciĂ³n se muestra lo mismo pero para una funciĂ³n cĂ³ncava. Fijaos que a medida que la variable \(x\) aumenta, la pendiente de las rectas tangentes, es decir, la derivada va disminuyendo. Es decir, las rectas tangentes van bajando. Se empieza con rectas tangentes de pendiente positiva, se llega a la recta tangente de pendiente cero (el mĂ¡ximo) y se acaba con rectas tangentes de pendiente negativa.

Concavidad y convexidad

Contenido bastante técnico.

DemostraciĂ³n del teorema

Demostremos la primera parte del teorema: supongamos que \(f''(x_0)>0\). Vamos a ver que \(f\) es convexa en un entorno de \(x_0\).

Como \(f''\) es continua al ser \(f\) tres veces derivable y \(f''(x_0)>0\), existe un \(\delta >0\) tal que \(f''(x)>0\), para todo \(x\in (x_0-\delta,x_0+\delta)\). Es decir, \(f''\) es positiva en un entorno del punto \(x_0\).

Sean \(x_1,x_2\in (x_0-\delta,x_0+\delta)\). Consideremos el desarrollo de Taylor de \(f\) en \(x_1\) de grado \(1\), es decir, el polinomio de Taylor en \(x_1\) de grado \(1\) junto con el resto de Lagrange y aplicamos dicho desarrollo a \(x_2\) \[ f(x_2) =f(x_1)+f'(x_1)\cdot (x_2-x_1)+\frac{f''(c_{12})}{2!}\cdot (x_2-x_1)^2, \] donde \(c_{12}\in <x_1,x_2>\subseteq (x_0-\delta,x_0+\delta)\).

Concavidad y convexidad

Como \(f''\) es positiva en el intervalo \((x_0-\delta,x_0+\delta)\), tendremos que \(f''(c_{12})>0\) y por tanto: \[ f(x_2)\geq f(x_1)+f'(x_1)\cdot (x_2-x_1), \] ya que \(\frac{f''(c_{12})}{2!}\cdot (x_2-x_1)^2>0\).

La condiciĂ³n anterior equivale a afirmar que \(f\) en convexa en el entorno de \(x_0\), \((x_0-\delta,x_0+\delta)\), usando la proposiciĂ³n anterior.

El caso \(f''(x_0)<0\) se razona de manera similar.

Demostremos la segunda parte del teorema: supongamos ahora que \(f''(x_0)=0\) y \(f'''(x_0)\neq 0\).

Supongamos para fijar ideas que \(f'''(x_0)>0\) (el caso \(f'''(x_0)<0\) se razonarĂ­a de forma similar).

Como \(\displaystyle f'''(x_0)=\lim_{x\to x_0}\frac{f''(x)-f''(x_0)}{x-x_0}=\lim_{x\to x_0}\frac{f''(x)}{x-x_0}>0\), tendremos que existe \(\delta >0\) tal que \(f''(x)>0\) si \(x\in (x_0,x_0+\delta)\) y \(f''(x)<0\) si \(x\in (x_0-\delta,x_0)\).

Concavidad y convexidad

Sean \(x_1,x_2\in (x_0-\delta,x_0)\). Si consideramos otra vez el desarrollo de Taylor de \(f\) en \(x_1\) de grado \(1\), y aplicamos dicho desarrollo a \(x_2\), obtenemos: \[ f(x_2) =f(x_1)+f'(x_1)\cdot (x_2-x_1)+\frac{f''(c_{12})}{2!}\cdot (x_2-x_1)^2, \] donde \(c_{12}\in <x_1,x_2>\subseteq (x_0-\delta,x_0)\).

Como \(f''\) es negativa en el intervalo \((x_0-\delta,x_0)\), tendremos que \(f''(c_{12})<0\) y por tanto: \[ f(x_2)\leq f(x_1)+f'(x_1)\cdot (x_2-x_1), \] ya que \(\frac{f''(c_{12})}{2!}\cdot (x_2-x_1)^2<0\). Es decir, \(f\) es cĂ³ncava en el intervalo \((x_0-\delta,x_0)\).

Concavidad y convexidad

Sean ahora \(x_1,x_2\in (x_0,x_0+\delta)\). Si consideramos otra vez el desarrollo de Taylor de \(f\) en \(x_1\) de grado \(1\), y aplicamos dicho desarrollo a \(x_2\), obtenemos: \[ f(x_2) =f(x_1)+f'(x_1)\cdot (x_2-x_1)+\frac{f''(c_{12})}{2!}\cdot (x_2-x_1)^2, \] donde \(c_{12}\in <x_1,x_2>\subseteq (x_0,x_0+\delta)\).

Como \(f''\) es positiva en el intervalo \((x_0,x_0+\delta)\), tendremos que \(f''(c_{12})>0\) y por tanto: \[ f(x_2)\geq f(x_1)+f'(x_1)\cdot (x_2-x_1), \] ya que \(\frac{f''(c_{12})}{2!}\cdot (x_2-x_1)^2>0\). Es decir, \(f\) es convexa en el intervalo \((x_0-\delta,x_0)\).

En resumen, acabamos de demostrar que \(x_0\) es un punto de inflexiĂ³n, tal como querĂ­amos ver.

Ejemplo

Ejemplo: estudio de la concavidad y convexidad de una funciĂ³n cĂºbica

Consideremos una funciĂ³ cĂºbica general dependiente de cuatro parĂ¡metros: \(f(x)=a x^3+b x^2+c x+d\), definida en todo el conjunto de nĂºmeros reales \(\mathbb{R}\), donde \(a,b,c\) y \(d\) son los parĂ¡metros de los que depende donde suponemos que \(a\neq 0\).

Usando el resultado anterior, estudiar la concavidad y convexidad equivale a estudiar el signo de la derivada segunda: \(f''(x)=2(3a x+b)\). Entonces:

  • si \(f''(x)>0\), o si \(x>-\frac{b}{3a}\), la funciĂ³n es convexa,
  • si \(f''(x)<0\), o si \(x<-\frac{b}{3a}\), la funciĂ³n es cĂ³ncava y,
  • si \(f''(x)=0\), o si \(x=-\frac{b}{3a}\), como \(f'''(x)=6a\neq 0\), \(f\) tiene un punto de inflexiĂ³n en dicho punto.

En el enlace siguiente se muestra la grĂ¡fica de la funciĂ³n \(f(x)=x^3-3 x^2+2x-1\), donde puede observarse que es convexa en la regiĂ³n \(\left(-\frac{b}{3a},\infty\right)=\left(-\frac{-3}{3\cdot 1},\infty\right)=(1,\infty)\), cĂ³ncava en la regiĂ³n \((-\infty,1)\) y tiene un punto de inflexiĂ³n en el punto \((1,-1)\).

Estudio local de una funciĂ³n

Una de las aplicaciones mĂ¡s usadas del estudio de las derivadas de primer y segundo orden es la representaciĂ³n aproximada de funciones.

Las derivadas son una herramienta muy Ăºtil para calcular las regiones donde la funciĂ³n crece y decrece, donde la funciĂ³n es convexa y cĂ³ncava asĂ­ como determinar los extremos relativos (mĂ¡ximos y mĂ­nimos) y los puntos de inflexiĂ³n.

Sin embargo, aparte del crecimiento y la concavidad, de cara a poder realizar una representaciĂ³n aproximada de la grĂ¡fica de una funciĂ³n, necesitamos estudiar otras propiedades.

Estudio local de una funciĂ³n

Concretamente, dichas propiedades son las siguientes:

  • DeterminaciĂ³n del dominio de la funciĂ³n.
  • CĂ¡lculo del conjunto formado por sus puntos de discontinuidad.
  • CĂ¡lculo de los puntos de corte con el eje de abscisas o eje de las \(x\) y el de ordenadas o eje de las \(y\).
  • Estudio de las simetrĂ­as con respecto el eje de ordenadas o eje de las \(y\) y respecto con el origen \((0,0)\).
  • CĂ¡lculo de las posibles asĂ­ntotas.
  • CĂ¡lculo de los extremos relativos, es decir, los mĂ¡ximos y mĂ­nimos de la funciĂ³n asĂ­ como los intervalos de crecimiento y decrecimiento de la misma.
  • CĂ¡lculo de los puntos de inflexiĂ³n de la funciĂ³n asĂ­ como los intervalos de convexidad y concavidad de la misma.

DeterminaciĂ³n del dominio de la funciĂ³n

Recordemos que el dominio de una funciĂ³n es el subconjunto de \(\mathbb{R}\) donde estĂ¡ definida.

Ejemplo

Consideremos la funciĂ³n \(f(x)=\frac{x^3-3 x+2}{x^3-x}\).

Al ser una funciĂ³n racional (cociente de dos polinomios), el dominio de la misma serĂ¡n todos los valores reales excepto aquĂ©llos que anulen el denominador: \[ x^3-x=0,\ \Leftrightarrow x\cdot (x^2-1)=0,\ \Leftrightarrow x=0,\ x^2-1=0,\ \Leftrightarrow x=0,\ x=\pm 1. \] Por tanto, el dominio de \(f\) serĂ¡ el conjunto siguiente: \(\mathbb{R}-\{-1,0,1\}\).

CĂ¡lculo del conjunto formado por sus puntos de discontinuidad

Recordemos que el conjunto de puntos de discontinuidad de una funciĂ³n son aquellos puntos de su dominio en los que la funciĂ³n no es continua.

Ejemplo (continuaciĂ³n)

En el ejemplo anterior en donde \(f(x)=\frac{x^3-3 x+2}{x^3-x}\), no hay puntos de discontinuidad ya que los Ăºnicos puntos en los que \(f\) no es continua serĂ­an aquĂ©llos en los que el denominador se anula pero Ă©stos no pertenecen al dominio.

CĂ¡lculo de los puntos de corte

Para calcular los puntos de corte de una funciĂ³n \(f(x)\) se tienen que realizar los procedimientos siguientes:

  • Puntos de corte con el eje de abscisas o eje de las \(x\). Hay que resolver la ecuaciĂ³n \(f(x)=0\). Sean (en caso que existan) \(x_1,\ldots,x_n\) sus soluciones. Entonces los puntos de corte son \((x_1,0),\ldots (x_n,0)\).

  • Punto de corte con el eje de ordenadas o eje de las \(y\). En este caso, sĂ³lo hay un punto de corte y es el siguiente: \((0,f(0))\).

CĂ¡lculo de los puntos de corte

Ejemplo (continuaciĂ³n)

Los puntos de corte para la funciĂ³n \(f(x)=\frac{x^3-3 x+2}{x^3-x}\) son los siguientes:

  • Eje de abscisas o eje X. Hay que resolver la ecuaciĂ³n siguiente: \(\frac{x^3-3 x+2}{x^3-x}=0\), que equivale a resolver \(x^3-3 x+2=0\).
Una soluciĂ³n de la ecuaciĂ³n anterior es \(x=1\). Para hallar las demĂ¡s, podemos descomponer el polinomio anterior usando la regla de Ruffini:
\(1\) \(0\) \(-3\) \(2\)
\(1\) \(1\) \(1\) \(-2\)
\(1\) \(1\) \(-2\) \(0\)

CĂ¡lculo de los puntos de corte

Las demĂ¡s soluciones serĂ¡n los ceros del polinomio \(x^2+x-2=0\): \[ x=\frac{-1\pm\sqrt{1+4\cdot 2}}{2}=\frac{-1\pm 3}{2}=1,-2. \] Los puntos de corte con el eje de abscisas o eje \(X\) son los siguientes: \((-2,0)\) y \((1,0)\).

  • El punto de corte con el eje de ordenadas o eje \(Y\) no existe en este caso ya que el valor \(x=0\) no es del dominio de la funciĂ³n tal como hemos indicado anteriormente.

Estudio de las simetrĂ­as

Averiguar si una funciĂ³n tiene simetrĂ­as es muy importante de cara a realizar una representaciĂ³n de la misma ya que nos dice cuĂ¡l es el valor de \(f(-x)\) a partir del valor de \(f(x)\) para cualquier valor de \(x\) de su dominio.

Existen dos tipos de simetrĂ­as:

  • SimetrĂ­a con respecto al eje de ordenadas o eje de las \(y\).
  • SimetrĂ­a con respecto al origen de coordenadas \((0,0)\).

Estudio de las simetrĂ­as

DefiniciĂ³n de simetrĂ­a con respecto al eje de ordenadas o eje de las \(y\).

Dada \(f\) una funciĂ³n real de variable real con dominio \(D\), diremos que tiene simetrĂ­a con respecto al eje \(Y\) si para cualquier valor \(x\) del dominio \(D\), \(-x\) tambiĂ©n estĂ¡ en \(D\) y ademĂ¡s \(f(-x)=f(x)\).

DefiniciĂ³n de simetrĂ­a con respecto al origen de coordenadas \((0,0)\).

Dada \(f\) una funciĂ³n real de variable real con dominio \(D\), diremos que tiene simetrĂ­a con respecto al origen de coordenadas \((0,0)\) si para cualquier valor \(x\) del dominio \(D\), \(-x\) tambiĂ©n estĂ¡ en \(D\) y ademĂ¡s \(f(-x)=-f(x)\).

Estudio de las simetrĂ­as

ObservaciĂ³n:

La Ăºnica funciĂ³n real de variable real que tiene los dos tipos de simetrĂ­as es la funciĂ³n idĂ©nticamente nula \(f(x)=0\) ya que si suponemos que \(f\) tiene los dos tipos de simetrĂ­as, tenemos por un lado que \(f(-x)=f(x)\) y por otro, que \(f(-x)=-f(x)\).

En resumen \(f(x)=-f(x)\), de donde deducimos que \(f(x)=0\), para todo valor \(x\) de su dominio.

Estudio de las simetrĂ­as

Ejemplo (continuaciĂ³n)

Veamos si la funciĂ³n \(f(x)=\frac{x^3-3 x+2}{x^3-x}\) tiene algĂºn tipo de simetrĂ­a:

El valor de \(f(-x)\) vale: \[ f(-x)=\frac{(-x)^3-3\cdot (-x)+2}{(-x)^3-(-x)}=\frac{-x^3+3x+2}{-x^3+x}, \] valor que, claramente es diferente de \(f(x)\) y de \(f(-x)\). Por tanto, la funciĂ³n \(f(x)\) no presenta ningĂºn tipo de simetrĂ­a.

Estudio de las simetrĂ­as

Comprobémoslo con un valor concreto de \(x\) de su dominio en python, por ejemplo \(x=2\):

def f(x):
 return((x**3-3*x+2)/(x**3-x))
 
f(2.)
## 0.6666666666666666
f(-2.)
## -0.0

CĂ¡lculo de las posibles asĂ­ntotas

Una asĂ­ntota es una recta que se aproxima a la funciĂ³n de forma que la distancia entre dicha funciĂ³n y la asĂ­ntota tiende a cero en un lĂ­mite determinado.

Su determinaciĂ³n es clave a la hora de realizar una representaciĂ³n de la funciĂ³n ya que nos dice como se comporta dicha funciĂ³n en lugares remotos.

Existen tres tipos de asĂ­ntotas:

CĂ¡lculo de las posibles asĂ­ntotas

AsĂ­ntotas horizontales. Diremos que la recta horizontal \(y=b\) para un \(b\) determinado es una asĂ­ntota horizontal de la funciĂ³n \(y=f(x)\) si, y sĂ³lo si, \(\displaystyle\lim_{x\to\pm \infty}f(x)=b\).

Es decir, \(\displaystyle\lim_{x\to\pm\infty} (f(x)-b)=0\). En este caso, se dice que la recta \(y=b\) es tangente a la funciĂ³n \(f(x)\) en el infinito. En la grĂ¡fica siguiente vemos una funciĂ³n (en verde) que tiene por un lado, cuando \(x\to \infty\) la asĂ­ntota horizontal \(y=1\) (en rojo) y, por otro, cuando \(x\to -\infty\) la asĂ­ntota horizontal \(y=-1\) (en azul).

CĂ¡lculo de las posibles asĂ­ntotas

CĂ¡lculo de las posibles asĂ­ntotas

AsĂ­ntotas verticales. Diremos que la recta vertical \(y=a\) para un \(a\) determinado es una asĂ­ntota vertical de la funciĂ³n \(y=f(x)\) si, y sĂ³lo si, \(\displaystyle\lim_{x\to a}f(x)=\pm \infty\).

En este caso, se dice que la recta \(x=a\) es tangente a la funciĂ³n \(f(x)\) en el infinito. En la grĂ¡fica siguiente vemos una funciĂ³n (en verde) que tiene una asĂ­ntota vertical en \(x=1\) (en rojo). Observar que: \[ \lim_{x\to 1^-} f(x)=\infty, \quad\lim_{x\to 1^+} f(x)=-\infty. \]

ObservaciĂ³n:

Usualmente, los posibles valores de \(a\) son puntos que no pertenecen al dominio de la funciĂ³n \(f\). Por tanto, los puntos de discontinuidad de la funciĂ³n son buenos candidatos para hallar dichos valores de \(a\).

CĂ¡lculo de las posibles asĂ­ntotas

CĂ¡lculo de las posibles asĂ­ntotas

AsĂ­ntotas oblicuas. Diremos que la recta vertical \(y=m x+n\) para unos \(m\) y \(n\) determinados es una asĂ­ntota oblicua de la funciĂ³n \(y=f(x)\) si, y sĂ³lo si, \(\displaystyle\lim_{x\to \pm\infty}(f(x)-(mx+n))=0\).

En este caso, se dice que la recta \(y=m x+n\) es tangente a la funciĂ³n \(f(x)\) en el infinito.

CĂ¡lculo de las posibles asĂ­ntotas

Para hallar el valor de \(m\), observar que el lĂ­mite \(\displaystyle\lim_{x\to\pm\infty}\frac{f(x)-(mx+n)}{x}=0\) ya que el numerador tiende a \(0\) y el denominador, a \(\pm\infty\), y \(\frac{0}{\pm\infty}=0\). Operando el lĂ­mite anterior, obtenemos \(m\): \[ \begin{array}{rl} \displaystyle \lim_{x\to\pm\infty}\frac{f(x)-(mx+n)}{x} & =0,\displaystyle \ \Leftrightarrow \lim_{x\to\pm\infty}\left(\frac{f(x)}{x}-m-\frac{n}{x}\right)=0,\Leftrightarrow\\ \displaystyle \lim_{x\to\pm\infty}\frac{f(x)}{x}-m & =0,\Leftrightarrow\displaystyle m = \lim_{x\to\pm\infty}\frac{f(x)}{x}. \end{array} \]

CĂ¡lculo de las posibles asĂ­ntotas

Para hallar el valor de \(n\), basta usar que \(\displaystyle\lim_{x\to \pm\infty}(f(x)-(mx+n))=0\) y el valor de \(m\) hallado anteriormente: \[ \begin{array}{rl} \displaystyle \lim_{x\to \pm\infty}(f(x)-(mx+n)) & =0,\displaystyle \Leftrightarrow \lim_{x\to \pm\infty}(f(x)-mx)-n=0,\Leftrightarrow\\ n & =\displaystyle\lim_{x\to \pm\infty}(f(x)-mx). \end{array} \]

En la grĂ¡fica siguiente vemos una funciĂ³n (en verde) que tiene la asĂ­ntota oblicua en \(y=x\) (en rojo). En este caso, tendremos que los valores de \(m\) y \(n\) serĂ¡n \(m=1\) y \(n=0\). Se puede observar que: \[ \lim_{x\to\infty} (f(x)-x)=0,\quad \lim_{x\to -\infty} (f(x)-x)=0. \]

CĂ¡lculo de las posibles asĂ­ntotas

Ejemplo

Ejemplo (continuaciĂ³n)

Hallemos las posibles asĂ­ntotas para la funciĂ³n \(f(x)=\frac{x^3-3 x+2}{x^3-x}\):

  • AsĂ­ntotas horizontales. Para hallar las asĂ­ntotas horizontales, tenemos que hacer el lĂ­mite de la funciĂ³n cuando el valor de \(x\) tiende a \(\pm\infty\): \[ \lim_{x\to\pm \infty} f(x)=\lim_{x\to\pm \infty} \frac{x^3-3 x+2}{x^3-x}=1. \] Entoces la recta horizontal \(y=1\) serĂ­a una asĂ­ntota horizontal de nuestra funciĂ³n.

  • AsĂ­ntotas verticales. Como ya hemos indicado anteriormente, unos buenos candidatos para las posibles asĂ­ntotas verticales \(x=a\) serĂ­an los valores de \(a\) correspondientes a los puntos de discontinuidad. Recordemos que tenemos tres puntos de discontinuidad: \(-1,0,1\). Probemos con cada uno de ellos haciendo el lĂ­mite correspondiente:

Ejemplo

  • \(x=-1\): \(\displaystyle\lim_{x\to -1}f(x)=\lim_{x\to -1}\frac{x^3-3 x+2}{x^3-x}=\frac{4}{0}=\infty.\) De hecho para ser mĂ¡s explĂ­citos, tendrĂ­amos que escribir: \[ \lim_{x\to -1^-}\frac{x^3-3 x+2}{x^3-x}=-\infty,\quad \lim_{x\to -1^+}\frac{x^3-3 x+2}{x^3-x}=+\infty, \] ya que si \(x\to -1^-\), significa que \(x<-1\) pero estĂ¡ cerca de \(-1\). Veamos cuĂ¡l es su signo para el valor \(x=-1.001\) por ejemplo:
f(-1.001)
## -1997.0019980021598

Es decir, su valor tiende a \(-\infty\).

Ejemplo

Si hacemos \(x\to -1^+\), significa que \(x>-1\) pero estĂ¡ cerca de \(-1\). Veamos cuĂ¡l es su signo para el valor \(x=-0.999\) por ejemplo:

f(-0.999)
## 2003.0020020020036

Es decir, su valor tiende a \(+\infty\) tal como hemos indicado anteriormente. Por tanto, la recta vertical \(x=-1\) es una asĂ­ntota vertical.

  • \(x=0\): \(\displaystyle\lim_{x\to 0}f(x)=\lim_{x\to 0}\frac{x^3-3 x+2}{x^3-x}=\frac{2}{0}=\infty.\) De hecho para ser mĂ¡s explĂ­citos, tendrĂ­amos que escribir: \[ \lim_{x\to 0^-}\frac{x^3-3 x+2}{x^3-x}=+\infty,\quad \lim_{x\to 0^+}\frac{x^3-3 x+2}{x^3-x}=-\infty, \] Por tanto, la recta vertical \(x=0\) es una asĂ­ntota vertical.

Ejemplo

  • \(x=1\): \(\displaystyle\lim_{x\to 1}f(x)=\lim_{x\to 1}\frac{x^3-3 x+2}{x^3-x}=\frac{0}{0}.\) Como hemos obtenido una indeterminaciĂ³n, la resolvemos descomponiendo los polinomios del numerador y del denominador: \[ \lim_{x\to 1}\frac{x^3-3 x+2}{x^3-x} =\lim_{x\to 1}\frac{(x^2+x-2)\cdot (x-1)}{x\cdot (x-1)\cdot (x+1)}= \lim_{x\to 1}\frac{(x^2+x-2)}{x\cdot (x+1)}=\frac{0}{2}=0. \] Entonces, la recta \(x=1\) no serĂ­a una asĂ­ntota vertical.

  • AsĂ­ntotas oblicuas. Existe la creencia sin justificar que dice que si una funciĂ³n tiene asĂ­ntotas horizontales, entonces no tiene asĂ­ntotas oblicuas. La razĂ³n de dicha creencia es que si \(\displaystyle\lim_{x\to \pm \infty}f(x)=b\neq 0\), entonces el valor de la pendiente \(m\) en la posible asĂ­ntota oblicua serĂ­a \(0\), siendo dicha asĂ­ntota una asĂ­ntota horizontal ya que: \(\displaystyle m=\lim_{x\to\pm\infty}\frac{f(x)}{x}=\frac{b}{\pm\infty}=0.\) Dicho razonamiento, aunque es vĂ¡lido la mayorĂ­a de las veces, tiene un pequeño problema en su lĂ­nea argumental. No podemos asegurar que no exista una funciĂ³n \(f(x)\) donde por ejemplo pase lo siguiente: \(\displaystyle\lim_{x\to +\infty}=b\) pero \(\displaystyle\lim_{x\to -\infty}\frac{f(x)}{x}=m\neq 0\). Es decir, que \(f(x)\) tiene una asĂ­ntota horizontal cuando \(x\) tiende a \(+\infty\) y una asĂ­ntota oblicua cuando \(x\to -\infty\).

Ejemplo

En resumen, aunque la mayorĂ­a de las veces no hace falta comprobar que haya asĂ­ntotas oblicuas si hay horizontales, hay que hacer la comprobaciĂ³n correspondiente.

En nuestro ejemplo, no hay asĂ­ntotas oblicuas ya que la pendiente correspondiente es \(0\): \[ m=\lim_{x\to\pm\infty}\frac{f(x)}{x}=\lim_{x\to\pm\infty}\frac{x^3-3 x+2}{x\cdot (x^3-x)}=\lim_{x\to\pm\infty}\frac{x^3-3 x+2}{x^4-x^2}=0. \]

Estudio del crecimento de la funciĂ³n

Para realizar una representaciĂ³n aproximada de una funciĂ³n, sĂ³lo es necesario conocer dos propiedades fundamentales: sus asĂ­ntotas y sus extremos relativos, es decir, sus mĂ¡ximos y mĂ­nimos relativos.

Seguidamente, para confirmar que nuestro esbozo es correcto, podemos determinar sus intervalos de crecimiento y decrecimiento de la misma y comprobar que los resultados obtenidos son los observados visualmente.

Vamos a dar un mĂ©todo prĂ¡ctico para realizar las dos tareas mencionadas anteriormente: el cĂ¡lculo de sus extremos relativos y los intervalos de crecimiento y decrecimiento.

Recordemos que una funciĂ³n crece en una regiĂ³n determinada de \(\mathbb{R}\) si su derivada es positiva y decrece si su derivada es negativa.

Estudio del crecimento de la funciĂ³n

Recordemos tambiĂ©n que los extremos relativos de la funciĂ³n deben verificar que su derivada debe ser nula. El primer paso serĂ¡, pues, calcular los ceros de la ecuaciĂ³n \(f'(x)=0\) calculando asĂ­ los candidatos a extremos relativos.

Sean, pues, \(x_1,\ldots,x_n\) los ceros de la ecuaciĂ³n \(f'(x)=0\) que son los candidatos a extremos relativos.

Sean tambien \(y_1,\ldots, y_k\) los puntos de discontinuidad de la funciĂ³n \(f(x)\).

Consideremos el conjunto uniĂ³n de los conjuntos anteriores \(\{x_1,\ldots,x_n\}\cup \{y_1,\ldots,y_k\}\) y nombremos \(z_i\), \(i=1,\ldots,k+n\) a sus elementos suponiendo Ă©stos por orden creciente: \[ \{x_1,\ldots,x_n\}\cup \{y_1,\ldots,y_k\} =\{z_1 < \cdots <z_{k+n}\}. \]

Estudio del crecimento de la funciĂ³n

A continuaciĂ³n construĂ­mos la tabla siguiente:

\(x\) \(-\infty\) \(z_1\) \(z_2\) \(\ldots\) \(z_{k+n}\) \(\infty\)
\(y'\) \(\pm\) \(\pm\) \(\ldots\) \(\pm\) \(\pm\)
\(y\) \(\nearrow \searrow\) \(\nearrow \searrow\) \(\ldots\) \(\nearrow \searrow\) \(\nearrow \searrow\)

En la segunda fila indicamos el signo de la derivada en los intervalos correspondientes, esto es, escribimos \(+\) si el signo de la derivada en el intervalo \((-\infty,z_1)\) es positivo y escribimos \(-\) si es negativo. Y lo mismo para los intervalos siguientes \((z_1,z_2)\) hasta llegar a \((z_{k+n},\infty)\). Para saber el signo de la derivada en un intervalo cualquiera, digamos \((z_{i-1},z_i)\), elegimos un valor \(z\) que pertenezca a dicho intervalo, por ejemplo su punto medio \(z=\frac{z_{i-1}+z_i}{2}\) y calculamos \(f'(z)\) para saber su signo.

Estudio del crecimento de la funciĂ³n

En la tercera fila, dependiendo del signo de la derivada, indicamos el comportamiento de la funciĂ³n escribiendo el signo \(\nearrow\) si la derivada es positiva \(+\) y escribiendo el signo \(\searrow\) si la derivada es negativa \(-\).

De esta manera si en un valor \(z_i\), a la izquierda del mismo aparece el signo \(\nearrow\) y a la derecha, el signo \(\searrow\) observĂ¡ndose \(\nearrow \searrow\) sabemos que en \(z_i\) hay un mĂ¡ximo relativo. Si en cambio es al revĂ©s, es decir, a la izquierda del mismo aparece el signo \(\searrow\) y a la derecha, el signo \(\nearrow\) observĂ¡ndose \(\searrow \nearrow\), sabemos que en \(z_i\) hay un mĂ­nimo relativo.

Los intervalos de crecimiento de la funciĂ³n \(f\) serĂ¡n aquĂ©llos en los que aparece el signo \(\nearrow\) y los intervalos de decrecimiento serĂ¡n aquĂ©llos en los que aparece el signo \(\searrow\).

Ejemplo

Ejemplo (continuaciĂ³n)

Para hallar los extremos relativos y los intervalos de crecimiento y decrecimiento de la funciĂ³n \(\frac{x^3-3 x+2}{x^3-x}\), tal como hemos indicado, resolvemos la ecuaciĂ³n \(f'(x)=0\).

En primer lugar, calculemos la funciĂ³n derivada \(f'(x)\): \[ \begin{array}{rl} f'(x) & =\frac{(3x^2-3)\cdot (x^3-x)-(3x^2-1)\cdot (x^3-3x+2)}{(x^3-x)^2}=\frac{3 x^5-6 x^3+3 x-(3 x^5-10 x^3+6 x^2+3 x-2)}{(x^3-x)^2}\\ & = \frac{4 x^3-6 x^2+2}{(x^3-x)^2}=\frac{(4x+2)\cdot (x-1)^2}{x^2\cdot (x-1)^2\cdot (x+1)^2}=\frac{4x+2}{x^2\cdot (x+1)^2}. \end{array} \]

Para resolver la ecuaciĂ³n \(f'(x)=0\), hemos de igualar a cero el numerador de la expresiĂ³n anterior \(4x+2=0\), obteniendo \(x=-\frac{2}{4}=-\frac{1}{2}\).

Ejemplo

Recordemos que los puntos de discontinuidad de la funciĂ³n \(f\) eran \(-1,0,1\). El conjunto de los \(z_i\) serĂ¡ en nuestro caso: \(\left\{-1,-\frac{1}{2},0,1\right\}\).

A continuaciĂ³n construimos la tabla anterior para calcular los intervalos de crecimiento y decrecimiento y los mĂ¡ximos y mĂ­nimos relativos de la funciĂ³n:

\(x\) \(-\infty\) \(-1\) \(-\frac{1}{2}\) \(0\) \(1\) \(\infty\)
\(y'\) \(-\) \(-\) \(+\) \(+\) \(+\)
\(y\) \(\searrow\) \(\searrow\) \(\nearrow\) \(\nearrow\) \(\nearrow\)

Ejemplo

Observando la tabla anterior, concluimos que

  • hay un mĂ­nimo relativo en el punto \(\left(-\frac{1}{2},f\left(-\frac{1}{2}\right)\right)=\left(-\frac{1}{2},9\right)\),
  • la funciĂ³n crece en la siguiente regiĂ³n de \(\mathbb{R}\): \(\left(-\frac{1}{2},0\right)\cup (0,1)\cup (1,\infty)\) y,
  • la funciĂ³n decrece en la siguiente regiĂ³n de \(\mathbb{R}\): \((-\infty,-1)\cup \left(-1,-\frac{1}{2}\right)\).

Concavidad y convexidad

Saber en quĂ© regiones una funciĂ³n es cĂ³ncava o convexa, ayuda de forma significativa a su representaciĂ³n.

Vamos a dar un mĂ©todo parecido al estudio del crecimiento y decrecimiento de una funciĂ³n cambiando los papeles de \(f'\) por \(f''\).

Recordemos que una funciĂ³n es convexa en una regiĂ³n de \(\mathbb{R}\) si su derivada segunda es positiva y es convexa, si es negativa.

Recordemos tambiĂ©n que los puntos de inflexiĂ³n deben verificar que su derivada segunda debe ser nula. El primer paso serĂ¡, pues, calcular los ceros de la ecuaciĂ³n \(f''(x)=0\) calculando de esta manera los candidatos a puntos de inflexiĂ³n.

Concavidad y convexidad

Sean \(\hat{x}_1,\ldots,\hat{x}_m\) los ceros de la ecuaciĂ³n \(f''(x)=0\) que son los candidatos a los puntos de inflexiĂ³n.

Recordemos que \(y_1,\ldots,y_k\) eran los puntos de discontinuidad de la funciĂ³n \(f(x)\).

Igual que antes, consideramos el conjunto uniĂ³n de los conjuntos \(\{\hat{x}_1,\ldots,\hat{x}_m\}\cup \{y_1,\ldots,y_k\}\) y nombramos \(\hat{z}_1,\ldots,\hat{z}_{k+m}\) a sus elementos suponiendo Ă©stos por orden creciente: \[ \{\hat{x}_1,\ldots,\hat{x}_m\}\cup \{y_1,\ldots,y_k\}=\{\hat{z}_1 < \cdots <\hat{z}_{k+m}\}. \]

Concavidad y convexidad

A continuaciĂ³n construĂ­mos la tabla siguiente:

\(x\) \(-\infty\) \(\hat{z}_1\) \(\hat{z}_2\) \(\ldots\) \(\hat{z}_{k+m}\) \(\infty\)
\(y''\) \(\pm\) \(\pm\) \(\ldots\) \(\pm\) \(\pm\)
\(y\) \(\bigcup \bigcap\) \(\bigcup \bigcap\) \(\ldots\) \(\bigcup \bigcap\) \(\bigcup \bigcap\)

En la segunda fila indicamos el signo de la derivada segunda en los intervalos correspondientes, esto es, escribimos \(+\) si el signo de la derivada en el intervalo \((-\infty,\hat{z}_1)\) es positivo y escribimos \(-\) si es negativo. Y lo mismo para los intervalos siguientes \((\hat{z}_1,\hat{z}_2)\) hasta llegar a \((\hat{z}_{k+m},\infty)\). Para saber el signo de la derivada en un intervalo cualquiera, digamos \((z_{i-1},z_i)\), elegimos un valor \(z\) que pertenezca a dicho intervalo, por ejemplo su punto medio \(z=\frac{z_{i-1}+z_i}{2}\) y calculamos \(f''(z)\) para saber su signo.

Concavidad y convexidad

En la tercera fila, dependiendo del signo de la derivada segunda, indicamos el comportamiento de la funciĂ³n escribiendo el signo \(\bigcup\) si la derivada segunda es positiva \(+\) y escribiendo el signo \(\bigcap\) si la derivada segunda es negativa \(-\).

De esta manera si en un valor \(\hat{z}_i\), a la izquierda del mismo aparece el signo \(\bigcup\) y a la derecha, el signo \(\bigcap\), o viceversa observĂ¡ndose \(\bigcup \bigcap\), o \(\bigcap \bigcup\) sabemos que en \(z_i\) hay un punto de inflexiĂ³n.

Los intervalos de convexidad de la funciĂ³n \(f\) serĂ¡n aquĂ©llos en los que aparece el signo \(\bigcup\) y los intervalos de concavidad serĂ¡n aquĂ©llos en los que aparece el signo \(\bigcap\).

Ejemplo

Ejemplo (continuaciĂ³n)

Para hallar los puntos de inflexiĂ³n y los intervalos de convexidad y concavidad de la funciĂ³n \(\frac{x^3-3 x+2}{x^3-x}\), tal como hemos indicado, resolvemos la ecuaciĂ³n \(f''(x)=0\).

En primer lugar, calculemos la funciĂ³n derivada \(f''(x)\): \[ \begin{array}{rl} f''(x) & = \frac{4\cdot x^2\cdot (x+1)^2-(4x+2)\cdot (2x\cdot (x+1)^2+2x^2\cdot (x+1))}{x^4\cdot (x+1)^4}\\ & =\frac{4\cdot x\cdot (x+1)-(4x+2)\cdot (2 (x+1)+2x)}{x^3\cdot (x+1)^3}=\frac{-4(3 x^2+3 x+1)}{x^3\cdot (x+1)^3}. \end{array} \] Para resolver la ecuaciĂ³n \(f''(x)=0\), hemos de igualar a cero el numerador de la expresiĂ³n anterior \(3 x^2+3 x+1=0\), obteniendo \(x=\frac{-3\pm\sqrt{9-3\cdot 4}}{6}=\frac{-3\pm\sqrt{-3}}{6}\). No tiene soluciĂ³n real, por tanto, la funciĂ³n no tiene puntos de inflexiĂ³n.

Ejemplo

Recordemos que los puntos de discontinuidad de la funciĂ³n \(f\) eran \(-1,0,1\). El conjunto de los \(\hat{z}_i\) serĂ¡ en nuestro caso: \(\left\{-1,0,1\right\}\).

A continuaciĂ³n construimos la tabla anterior para calcular los intervalos de convexidad y concavidad de la funciĂ³n:

\(x\) \(-\infty\) \(-1\) \(0\) \(1\) \(\infty\)
\(y''\) \(-\) \(+\) \(-\) \(-\)
\(y\) \(\bigcap\) \(\bigcup\) \(\bigcap\) \(\bigcap\)

Observando la tabla anterior, concluimos que

  • la funciĂ³n es convexa en la siguiente regiĂ³n de \(\mathbb{R}\): \((-1,0)\) y
  • la funciĂ³n es cĂ³ncava en la siguiente regiĂ³n de \(\mathbb{R}\): \((-\infty,-1)\cup (0,1)\cup (1,\infty)\).

Ejemplo

Las propiedades que hemos estado hallando que en forma de resumen son las siguientes:

  • Corte con los ejes: \((-2,0)\) y \((1,0)\).
  • AsĂ­ntotas: \(y=1\), \(x=-1\) y \(x=0\).
  • MĂ­nimo en \(\left(-\frac{1}{2},9\right)\), \(f\) crece en \(\left(-\frac{1}{2},0\right)\cup (0,1)\cup (1,\infty)\) y decrece en \((-\infty,-1)\cup \left(-1,-\frac{1}{2}\right)\).
  • No tiene puntos de inflexiĂ³n, es convexa en \((-1,0)\) y cĂ³ncava en \((-\infty,-1)\cup (0,1)\cup (1,\infty)\).

En el grĂ¡fico siguiente hemos indicado las asĂ­ntotas (en rojo), los puntos de corte y el mĂ­nimo \((\bigcup)\). ¿SerĂ­ais capaces de acabar la representaciĂ³n grĂ¡fica?

Si querĂ©is ver el grĂ¡fico de la funciĂ³n que hemos desarrollado en Wolfram Alpha, id al enlace siguiente: .

Ejemplo